Neuro Anatomy

Download as pdf or txt
Download as pdf or txt
You are on page 1of 287

Preface

Agam is a group of budding medicos, who are currently doing their under graduation in
various Medical Colleges across Tamil Nadu and Pondicherry. The group was initiated on 18th
November 2017, in the vision of uniting medicos for various social and professional causes.

We feel delighted to present you Agam Anatomy notes prepared by Agam Divide and Rule
2020 Team to guide our fellow medicos to prepare for university examinations.

This is a reference work of 2017 batch medical students from various colleges. The team
took effort to refer many books and make them into simple notes. We are not the authors of the
following work. The images used in the documents are not copyrighted by us and is obtained from
various sources.

Dear readers, we request you to use this material as a reference note, or revision note, or
recall notes. Please do not learn the topics for the 1st time from this material, as this contain just the
required points, for revision.
Acknowledgement

On behalf of the team, Agam would like to thank all the doctors who taught us Anatomy. Agam
would like to whole heartedly appreciate and thank everyone who contributed towards the making
of this material. A special thanks to Srivardhany Bhaskar and M. Snaha, who took the responsibility
of leading the team. The following are the name list of the team who worked together, to bring out
the material in good form.
• Priyadharshini
• Mohanraj
• Sanjana Singh
• Yashi Awasthi
• Tharshna
• Ram Girithar
• Sudharshan
• Amrutha
• Ashiq
• BalaMurugan
• Prasanna
• Ajithvaas
• Shaziya
• Harsha M
• Barani
• Rhifkha
• Kiruthika
• Saranya
• Sakthi
• Geethik
• Thamizhazhagan
• Bharani
NEUROANATOMY QUESTION BANK

SR. PAGE.
Topic
NO. NO.

1. Spinal Cord 11

2. 6th Cranial Nerve 20

3. Facial Nerve 23

4. Trochlear Nerve 30

5. Glossopharyngeal Nerve 33

6. Oculomotor Nerve 38

7. Medulla Oblongata 42

8. Cerebellum 50

9. Hypothalamus 58

10. 3rd Ventricle 62

11. Superolateral Surface Of Cerebral Hemisphere 65

12. Arterial Supply of Cerebral Hemisphere: 72

13. White Matter of Cerebrum 76

14. Hypothalamus 81

Page 1 of 284
15. Nuclei Of Thalamus and Their Connections 86

16. Basal Ganglia 95

17. Meninges And Meningeal Spaces 102

18. Dural Folds 109

19. Falx Cerebri 115

20. Superior Sagittal Sinus 116

21. Cavernous Sinus 118

22. Oculomotor Nerve 121

23. Visual Pathway 123

24. Chorda Tympani 124

25. Trigeminal Ganglion 125

26. Foramen Magnum 131

27. Jugular Foramen 133

28. Pterion 134

29. Relations Of Cavernous Sinus 135

30. Sigmoid Sinus 137

Formation & Distribution of Spinal Part of Accessory


31. 138
Nerve:

Page 2 of 284
32. Hypoglossal Nerve 140

33. Third Ventricle of Brain 143

34. Metathalamus 146

35. Labelled Diagram of Pons at The Level of Facial Colliculus 148

36. Transverse Section Through the Upper Part of Pons 149

37. Pontocerebellar Angle Syndrome 150

38. Microscopic Structure of Cerebellum 150

39. Rhomboid Fossa 153

40. Cerebellar Peduncle 157

41. Interpeduncular Fossa 158

42. Dentate Nucleus 159

Enumerate The Important Signs and Symptoms Seen in


43. 160
Cerebellar Lesion

44. Lateral Medullary Syndrome: 161

45. Medial Medullary Syndrome 162

46. T.S. Of Mid-Brain at The Level of Inferior Colliculi 163

47. T. S. Of Midbrain at The Level of Superior Colliculi 165

48. Medial Longitudinal Bundle 168

Page 3 of 284
49. Mlf Syndrome (Internuclear Ophthalmoplegia) 170

50. Horns Of Lateral Ventricle 171

51. Fourth Ventricle 174

52. Medulla: Pyramidal Decussation 177

53. Medulla (Olivary Level) 179

Transverse Section Through Middle of Medulla Passing


54. 181
Through Sensory Decussation

55. Diencephalon- Definition and Subdivisions 183

56. Thalamus - Definition and Nuclei 185

57. Pineal Gland 187

58. Parts, Deep Nuclei & Arterial Supply of Cerebellum 189

59. Calcarine Sulcus 193

60. Connections Of Basal Ganglia 194

Labelled Diagram of Superolateral Surface of Cerebrum,


61. 197
Indicating Major Functional Areas

62. Structures Forming the Limbic System 198

63. Components Of Basal Ganglia 200

64. Parkinsonism 202

65. Functional Areas of Superior Temporal Gyrus. 202

Page 4 of 284
66. Name The Two Sensory Thalamic Nuclei 203

67. Insula 203

68. Superolateral Surface Of Cerebrum 205

69. Blood Supply of The Spinal Cord 209

70. Circle Of Willis (Circulus Arteriosus) 212

71. Blood Supply of Brain 219

72. Name Of the Bones Meeting at Pterion 227

73. Falx Cerebri 228

74. Epicranial Aponeurosis 230

75. Show The Venous Sinuses Related to Falx Cerebri 230

Name The Structures Passing Through Internal Acoustic


76. 230
Meatus

77. Name The Structure Passing Through Foramen Spinosum 231

Name The Structures Present in Lateral Wall of Cavernous


78. 231
Sinus

79. Pterion 233

80. List Out the Paired Dural Venous Sinuses 235

81. List Out the Unpaired Dural Venous Sinuses 236

82. Tentorium Cerebelli 237

Page 5 of 284
83. Suboccipital Nerve 239

84. Tributaries Of Cavernous Sinus 239

85. Structures Passing Through the Foramen Ovale 240

86. Ligamentum Denticulatum 240

87. Arachnoid Granulations 241

88. Cross Sections of Spinal Cord at Cervical Level 241

89. Cross Section of Spinal Cord at Thoracic Level 242

90. Lumbar Puncture 243

91. Nerves Rising from Medulla Oblongata 243

92. List Somatic Afferent Nuclei 244

93. Modifications Of Cranial Pia Matter 244

94. Terminal Branches of Facial Nerve 244

95. Indicate The Paleo Cerebellar Deep Nuclei 245

Deep Nuclei of Cerebellum (Also Called as Intracerebellar


96. 245
Nuclei or Central Nuclei)

97. List Special Somatic Afferent Nuclei 245

98. Middle Meningeal Artery 246

99. Pineal Gland 246

Page 6 of 284
100. Origin And Branches of Middle Meningeal Artery 247

101. Formation Of Basal Vein 248

102. Basilar Artery 248

103. Corpus Striatum 249

104. Structures Forming Limbic System 249

105. Components Of Basal Ganglia 250

106. Parkinsonism 251

107. Functional Areas of Superior Temporal Gyrus 251

108. Sensory Thalamic Nuclei 252

109. Insula 252

110. Visual Stria 252

111. Structures Lodged in Lateral Sulcus of Cerebrum 253

112. Parts Of Caudate Nucleus 253

113. Corpus Callosum 253

114. Parts Of Corpus Callosum 254

115. Blood Supply of Internal Capsule 254

116. Name Of the Bones Meeting at Pterion 255

Page 7 of 284
117. Falx Cerebri 256

118. Epicranial Aponeurosis 258

119. Show The Venous Sinuses Related to Falx Cerebri 258

Name The Structures Passing Through Internal Acoustic


120. 258
Meatus

121. Name The Structure Passing Through Foramen Spinosum 259

Name The Structures Present in Lateral Wall of Cavernous


122. 259
Sinus

123. Pterion 261

124. List Out the Paired Dural Venous Sinuses 263

125. List Out the Unpaired Dural Venous Sinuses 264

126. Tentorium Cerebelli 265

127. Suboccipital Nerve 267

128. Tributaries Of Cavernous Sinus 267

129. Structures Passing Through the Foramen Ovale 268

130. Ligamentum Denticulatum 268

131. Arachnoid Granulations 269

132. Cross Sections of Spinal Cord at Cervical Level 269

133. Cross Section of Spinal Cord at Thoracic Level 270

Page 8 of 284
134. Lumbar Puncture 271

135. Nerves Rising from Medulla Oblongata 271

136. List Somatic Afferent Nuclei 272

137. Modifications Of Cranial Pia Matter 272

138. Terminal Branches of Facial Nerve 272

139. Indicate The Paleo Cerebellar Deep Nuclei 273

Deep Nuclei of Cerebellum (Also Called as Intracerebellar


140. 273
Nuclei or Central Nuclei)

141. List Special Somatic Afferent Nuclei 273

142. Pineal Gland 274

143. Origin And Branches of Middle Meningeal Artery 275

144. Formation Of Basal Vein 276

145. Basilar Artery 276

146. Corpus Striatum 277

147. Structures Forming Limbic System 277

148. Components Of Basal Ganglia 278

149. Parkinsonism 279

150. Functional Areas of Superior Temporal Gyrus 279

Page 9 of 284
151. Sensory Thalamic Nuclei 280

152. Insula 280

153. Visual Stria 280

154. Structures Lodged in Lateral Sulcus of Cerebrum 281

155. Parts Of Caudate Nucleus 281

156. Corpus Callosum 281

157. Parts Of Corpus Callosum 282

158. Blood Supply of Internal Capsule 282

Page 10 of 284
SPINAL CORD
 Spinal cord is the main pathway for information connecting the
brain and the peripheral nervous system.
 It occupies the upper two third of the vertebral canal and gives
rise to 31 passes of spinal nerves.

FEATURES
 The spinal cord is 45 cm in adult male and 42 cm in adult female.
 It extends from the upper border of the Atlas vertebra to the lower
border of the first lumbar vertebra in adults.

MENINGEAL COVERINGS
 The spinal cord is surrounded by three meninges.
 The outermost is the dura matter the middle one is arachnoid
matter and the innermost is the Piamater
 The space between the dura mater and arachnoid mater is called
subdural space
 The arachnoid matter and diameter are separated by
subarachnoid space which contains the cerebrospinal fluid
 The space between the meningeal layer and the end ostium of the
vertebral canal is called epidural space, where epidural
anaesthesia can be given.
 The spinal Piamater undergoes modifications to keep the spinal
cord in position.

1. LIGAMENTA DENTICULATA
 21 pairs of teeth like projections
 Lately fuses with arachnoid and dura matters midway
between the exits of the roots of adjacent spinal nerves
 The highest process attaches immediately superior to
foramen Magnum

Page 11 of 284
 The ligamentum denticulate keeps the spinal cord in
position

2. LINEA SPLENDENS
 It is a thickening seen at the Antero median sulcus in the
lower part of the spinal cord

3. FILUM TERMINALE
 20 centimetres long
 Leaves through the sacral hiatus and ends by getting
attached to the periosteum of dorsal surface of first segment
of coccyx
 Consists of two parts
a) FILUM TERMINALE INTERNUM - is the upper
part which is 15 cm long, extends up to the lower
border of second sacral vertebra
b) FILUM TERMINALE EXTERNUM - logon part
which is attached to the first segment of the coccyx.
Between the lower border of L1 and S2 vertebrae the
subarachnoid space contains spinal nerves which
constitute the cauda equina

ENLARGEMENTS
 Neurones at appropriate levels form
enlargements to be able to supply increase
musculature
1. CERVICAL ENLARGEMENT
for supply of upper limb muscles -
extends from C4 to t1 spinal
segments with maximum
diameter of 38 millimetre at the
level of C6 segment

Page 12 of 284
2. LUMBAR ENLARGEMENTS for supply of muscles of
lower limb - extends from level of l2 to three segments. Its
maximum diameter is 35 millimetres at level of S1 segment

CAUDA EQUINA
 Dorsal and ventral roots of right and left sides of l2 to L5 S1 to S5
and Co1 nerves lies almost vertical around filum terminale.
 These are called cauda equina.
 There are 40 nerve roots at the beginning of cauda equina
 One dorsal and one ventral root joins to form one spinal nerve and
leaves through the intervertebral foramen on one side
 So, at every intervertebral foramen for nerve roots exit the cauda
equina leaving it thinner.

EXTERNAL FEATURES OF SPINAL CORD


 Anteriorly, the spinal cord reveals a deep anterior median fissure
lodging the anterior spinal artery.
 posterior median sulcus is a longitudinal groove from which a
septum runs in the depth of spinal cord
 Each half is subdivided into anterior later and posterior regions by
anterolateral and posterior lateral sulci
 Ventral are motor nerves emerge from the anterolateral sulcus.
 dorsal or sensory nerve roots and spinal cord from posterolateral
sulcus

Page 13 of 284
INTERNAL STRUCTURE
 White matter lies outside and grey matter lies inside
 In the centre of grey matter is the central canal containing the CSF
 The grey matter is in the form of H shape with the grey commissure
joining the right and left sides
 Each side of the grey matter contains one anterior horn and one
posterior horn.

Page 14 of 284
Page 15 of 284
SEGMENTS OF POSTERIOR LATERAL
ANTERIOR HORN
SPINAL CORD HORN HORN

Narrow in 1to 3
segments
Cervical, oval
Slender Absent Brought in C4 to c8
shape
segments for supply
of upper limb

Present for Slender in t2 to t12


Thoracic circular
Slender thoracolumbar segments brought in
shape
outflow t1 segment

Present only in
Lumbar circular Bulbous for supply of
Bulbous number one and
shape lower limbs
two segments

Group of cells in
Sacral circular butt cycle 2 to 4 Bulbous for supply of
Thick
smaller segments for lower limbs
sacral outflow

BLOOD SUPPLY OF SPINAL CORD


 It receives blood supply from three longitudinal arterial channels
that extend along the length of the cord.
 The anterior spinal artery is present in anterior median sulcus
 Two posterior spinal arteries run along the posterior lateral sulcus
 The main source of blood to spinal arteries is from the vertebral
arteries (up to the cervical segments)

Page 16 of 284
 it also receives blood from radicular arteries that reach the cord
along the roots of spinal nerves.
 Frequently one of the anterior radicular branches is very large
and is called arteria radicularis magna

VENOUS DRAINAGE
 The veins are arranged in the form of 6 longitudinal channels
 These are:
1. Anteromedian vein
2. Posteromedian vein
 Anterolateral and posterolateral channels that are paired
 These channels are interconnected by a plexus of veins that form a
venous vasocorona
 The blood from these veins is drain by radicular veins that open
into epidural and internal vertebral plexus.

Page 17 of 284
APPLIED ANATOMY
1. The medical procedure known as a lumbar puncture spinal tap
involves use of a needle to withdraw cerebrospinal fluid from the
sub arachnoid space usually from the lumbar region of the spine.
It is done at the lower level of L4 vertebra
2. CONUS MEDULLARIS SYNDROME: due to injury to s2-s4
segments of spinal cord
a. Sexual functions are affected as same as carry out sexual
functions as well
b. involvement of bladder and bowel is early as s2-S4 segments
carry Sacral component of the parasympathetic system
which supplies the bladder and lower bowel

Page 18 of 284
3. CAUDA EQUINA SYNDROME: damage to cauda equina results
in
a. Lower motor neurone type of paralysis in the lower limb due
to compression of ventral nerve roots
b. Bladder and bowel involvement is late.
c. Root pain is an important symptom due to involvement of
dorsal nerve roots
4. POLIOMYELITIS: it is a viral disease which involves anterior
horn cells leading to flaccid paralysis of the affected segment.

Page 19 of 284
6TH CRANIAL NERVE

INTRODUCTION:
 6TH cranial nerve
 Purely motor
 Supplies only one muscle-lateral rectus of eye
 Most susceptible to damage all the cranial nerves during the
increased intracranial pressure

FUNCTIONAL COMPONENTS AND NUCLEI:


 General somatic afferent fibres:
■ *Carry proprioceptive signals from lateral rectus
■ *Terminate in mesencephalic nucleus of trigeminal nerve
 General somatic efferent fibres:
■ *Arise from the abducent nucleus in pons
■ *Supply the lateral rectus muscle of the eyeball

COURSE, RELATIONS, DISTRIBUTIONS:


Arises at the lower border of the pons opposite to the pyramid of the
medulla.

runs upwards, forward and laterally dorsal to the anterior cerebellar
artery

pierces the duramater over the clivus inferolateral to the dorsum sellae

passes through the medial wall of the inferior petrosal sinus

Page 20 of 284

arches forward directly over the shape ridge of the petrous temporal
bone, under the petroclinoid ligament

Enters the fibro osseous or Dorello’s canal

Enters the cavernous sinus, by piercing the posterior wall close to the
floor of the sinus (course: runs forward inferolateral to the internal
carotid artery)

Through the superior orbital fissure

Enters the orbit, runs forward toward the lateral side to the orbital
surface of the lateral rectus muscle which it supplies

CLINICAL CORRELATION:
 Damaged during increased intracranial pressure
 Nerve is cut by the sharp bony edge of the petrous temporal bone
 Paralysis of lateral rectus muscle following the injury to abducents
nerve leads to convergent squint
1. Inability to abduct the eye
2. Diplopia (double vision)

Page 21 of 284
Page 22 of 284
FACIAL NERVE
 Seventh cranial nerve
 Mixed nerve (mainly motor)
 Nerve of second pharyngeal arch
 Most frequently paralysed peripheral nerve of body

FUNCTIONAL COMPONENTS AND NUCLEI OF

Page 23 of 284
FACIAL NERVE

SPECIAL VISCERAL EFFERENT:


 Motor nucleus of facial nerve in pons
 Supplies muscles of facial expression

GENERAL VISCERAL EFFERENT:


 Superior salivatory nucleus & lacrimatory nucleus
 Secretomotor fibres to lacrimal glands and mucous glands of nasal
cavity and palate

SPECIAL VISCERAL AFFERENT:


 Nucleus tractus solitarius
 Carry taste sensations from anterior 2/3rd of tongue except vallate
papillae

GENERAL SOMATIC AFFERENT:


 Spinal nucleus of Trigeminal nerve
 Carry general sensation from skin of auricle

ORIGIN
 Emerges from pontomedullary junction
 2 roots
1. large motor root
2. small sensory root (nervus intermedius)
 Relations- sensory root lies between motor root medially and
vestibulocochlear nerve laterally

Page 24 of 284
COURSE
 Facial nerve has both intracranial and extracranial course.

Page 25 of 284
TERMINATION
 Within parotid gland, facial nerve divides into 5 terminal branches
supplying muscles of facial expression.

BRANCHES
INTRACRANIAL BRANCHES:
a) GREATER PETROSAL NERVE
 First branch of facial nerve
 Arises from geniculate ganglion
 Supplies lacrimal glands, mucous glands of nasal cavity and
palate
b) NERVE TO STAPEDIUS
 Arises from vertical part of facial nerve
 Supplies stapedius muscle
c) CHORDA TYMPANI
 Arises from vertical part of facial nerve 6 mm above
stylomastoid foramen
 Joins with lingual nerve, via submandibular ganglion supplies
submandibular and sublingual glands
 Carry taste sensations from anterior 2/3rd of tongue except
vallate papillae
d) EXTRACRANIAL BRANCHES
1. Posterior auricular nerve – supplies occipitalis muscle
2. Nerve to posterior belly of digastric
3. Nerve to stylohyoid
4. 5 terminal branches
5. Temporal branch
6. Zygomatic branch
7. Buccal branch
8. Marginal mandibular branch
9. Cervical branch

Page 26 of 284
Page 27 of 284
APPLIED ASPECT
 Paralysis of stapedius – Hyperacusis
 Injury to greater petrosal nerve – Loss of lacrimation
 Lower motor neuron lesion - Bell's palsy
 Usual site of lesion: at or just below stylomastoid foramen

Page 28 of 284
CLINICAL FEATURES OF BELL'S PALSY
 All muscles of facial expression are paralysed
1. LOSS OF TRANSVERSE WRINKLES ON FOREHEAD –
paralysis of frontalis
2. INABILITY TO CLOSE EYE – paralysis of orbicularis oculi
3. OVERFLOW OF TEARS ON CHEEK
4. DROOLING SALIVA FROM ANGLE OF MOUTH – due to
paralysis of levator anguli Oris, orbicularis Oris
5. ACCUMULATION OF FOOD IN VESTIBULE OF MOUTH
– paralysis of buccinator muscle
6. FACIAL ASYMMETRY – Face pulled to healthy side

CROCODILE TEARS SYNDROME


 Paroxysmal lacrimation during eating
 Facial nerve lesion proximal to geniculate ganglion
 Regenerating preganglionic fibres meant to provide secretomotor
supply to submandibular and sublingual salivary glands, during
regeneration misdirected to grow in endoneural sheaths of
preganglionic secretomotor fibres which supply lacrimal gland

RAMSAY HUNT SYNDROME


 Involvement of geniculate ganglion in herpes zoster infection
1. Herpetic vesicles on auricle
2. Hyperacusis
3. Loss of lacrimation
4. Loss of taste in anterior 2/3 rd of tongue
5. Complete ipsilateral facial palsy

Page 29 of 284
TROCHLEAR NERVE – 4th CRANIAL NERVE

 Supplies only superior oblique muscle of the eyeball.


 Origin -Only cranial nerve - emerge on the dorsal aspect of the
eyeball.

FUNCTIONAL COMPONENT
1. GENERAL SOMATIC EFFERENT FIBRES- for lateral movement
of eyeball.
2. GENERAL SOMATIC AFFERENT FIBRES -proprioceptive
impulse, from muscle to mesencephalic nucleus of 5th nerve.

NUCLEUS
 Present in the ventromedial part of central grey matter of midbrain
at the level of inferior colliculus.
 Related close to medial longitudinal bundle (ventrally).

Page 30 of 284
COURSES
1. Intra neural course - runs dorsally round the central grey matter
to reach upper part of anterior medullary velum, decussate to
opposite nerve.
2. It attached to anterior medullary velum, one on each side of
frenulum veli (just below inferior colliculus)

3. Wind around superior cerebellar peduncle and cerebral peduncle,


pass between posterior cerebral artery and superior cerebellar
Page 31 of 284
artery, then appear ventral between temporal lobe and upper
border of pons
4. Enter cavernous sinus-pierces roof and runs lateral wall, between
oculomotor and ophthalmic nerve.
5. Enter the orbit -enter through lateral part of superior orbital
fissure.
6. In orbit-passes medially, lies over levator palpebrae superioris
then it supplies superior oblique muscle.

APPLIED
WHEN TROCHLEAR NERVE - DAMAGED
 Diplopia occurs, looking downward, vision is single (above the
horizontal plane).
WHEN TROCHLEAR NERVE - PARALYSIS
 Diplopia
 Defective depression of adducted eye.

Page 32 of 284
GLOSSOPHARYNGEAL NERVE:

INTRODUCTION:
 Glossopharyngeal Nerve is the ninth cranial nerve.
 It is a mixed nerve, that is composed of both the motor and sensory
fibres, but predominantly it is sensory.

FUNCTIONAL COMPONENTS AND NUCLEI:

Page 33 of 284
1. SVE fibres: They supply stylopharyngeus muscle. They arise
from nucleus ambiguous.
2. GVE fibres: They supply the secretomotor fibres to the parotid
gland. They are preganglionic parasympathetic fibres and arise
from inferior salivatory nucleus.
3. SVA fibres: They carry taste sensations from the posterior one
third of tongue including vallate papillae and terminate in the
nucleus tractus solitarius.
4. GVA fibres: They carry general sensations of pain, touch and
temperature from the mucous membrane of the pharynx, tonsil,
soft palate, and the posterior one-third of tongue and terminate in
the dorsal nucleus of the vagus.
5. GSA fibres: They carry proprioceptive sensations from
stylopharyngeus and skin of the auricle and terminate in the
nucleus of the spinal tract of the 5th Nerve.

ORIGIN AND COURSE:


 They arise from the pontomedullary junction from behind the olive
of Medulla by three or four rootlets. They unite to form a single
trunk. It leaves the cranial cavity by passing through jugular
foramen.
 Small superior Ganglion and larger inferior ganglion located
within the jugular foramen and at the base of the skull, the nerve
passes downward and forward between the internal carotid artery
and internal jugular vein.
 It then descends anterior to the internal carotid artery to styloid
process and muscles attached to it to reach the lower border of the
stylopharyngeus through the gap between the superior and middle
constrictors of the pharynx.
 It then passes deep to the stylohyoid ligament and posterior edge
of the hyoglossus muscle, here it breaks up into terminal
branches, which supply the mucous membrane of posterior one
third of tongue, pharynx and tonsil.

Page 34 of 284
Page 35 of 284
BRANCHES AND DISTRIBUTION:
TYMPANIC BRANCH (JACOBSON'S NERVE):
 It leaves inferior ganglion and enters via bony edge between jugular
foramen and carotid canal. It forms the tympanic plexus over middle
ear promontory.
 It gives off
a) The lesser petrosal nerve
b) Twigs it tympanic cavity, auditory tube, and mastoid air
cells.
 The lesser petrosal nerve carries the preganglionic parasympathetic
fibres which relay in the otic ganglion. The parasympathetic fibres
from the ganglion supply the parotid gland.

CAROTID NERVE (HERRING'S NERVE): It is a branch to carotid


sinus and body - afferent for pressoreceptor and chemoreceptor reflexes
to regulate heart rate and respiration, respectively.

PHARYNGEAL BRANCH: It joins the pharyngeal branches of vagus


and cervical sympathetic chain to form pharyngeal plexus on middle
constrictor of pharynx.

BRANCH TO STYLOPHARYNEGEUS: It arises as nerve, winds


around the stylopharyngeus muscle. It is the only motor branch of
Glossopharyngeal Nerve.

TONSILLAR BRANCHES: They supply the mucous membrane of


tonsil, faces and palate.

LINGUAL BRANCHES: They supply the posterior one third of the


tongue and vallate Papillae and convey taste and general sensations.

Page 36 of 284
APPLIED ANATOMY:

LESION OF GLOSSOPHARYNGEAL NERVE: The lesion is rare in


isolated so in love along with vagus nerve lesion. Complete lesion causes
a) Swallowing Difficulty
b) Unilateral gag reflex loss
c) Loss of parotid salivation
d) Loss of taste and general sensations over post-1/3rd of tongue.

FREY'S SYNDROME / AURICULOTEMPORAL NERVE


SYNDROME:
 Due to injury to Auriculotemporal Nerve during parotidectomy.
 Gustatory sweating occurs occur while taking food

GLOSSOPHARYNGEAL NEURALGIA: pain in area of sensory


distribution of Glossopharyngeal nerve.

CLINICAL TESTING OF THE NERVE:


a) By eliciting gag reflex.
b) Testing the taste sensations in the posterior one third of the
tongue

CONCLUSION:
 This nerve has motor supply to only one muscle (stylopharyngeus)
and rest all are sensory supply.

Page 37 of 284
OCULOMOTOR NERVE
 It is the 3rd cranial nerve. It is somatic motor nerve distributed to
extraocular & intraocular muscles.

Page 38 of 284
ORIGIN
 The fibres arise from the oculomotor nucleus.
 From the mid brain at the level of superior colliculi.

COURSE
INTRANEURAL COURSE:
 Fibres arise from nucleus.
 Pass ventrally through ___
a) Tegmentum.
b) Red nucleus.
c) Substantia nigra.

INTRACRANIAL COURSE:
 Attached to oculomotor sulcus on medial side of crus cerebri at the
base of brain.
 Runs forward in the interpeduncular cistern.
 Reaches cavernous sinus & enters by piercing the posterior part of
roof on lateral side of the posterior clinoid process.
 Lies in lateral wall of sinus above trochlear nerve.
 At the anterior part divides into small upper and large lower
divisions.
 Leaves the cranium by passing in middle part of the superior
orbital fissure.

IN THE ORBIT:
 Upper division ascends on lateral of optic nerve.
 Lower division divides into 3 branches.

RELATIONS
 After emerging from mid brain lies between superior cerebellar &
posterior cerebral arteries.
Page 39 of 284
 In interpeduncular cistern lies on the lateral side of posterior
communicating artery.
 In lateral wall of sinus lies above the trochlear nerve.
 In the fissure, the nasociliary nerve lies in between the two
divisions.
 Abducent nerve lies inferolateral to them.

BRANCHES
SMALL UPPER DIVISION:
 Supplies superior rectus & part of levator palpebrae superioris.
LARGE LOWER DIVISION:
a) Nerve to medial rectus.
b) Nerve to inferior rectus.
c) Nerve to inferior oblique (long).
1. Parasympathetic root for ciliary ganglion.
2. Supplies inferior oblique.

NUCLEUS
OCULOMOTOR NUCLEUS:
 Ventromedial part of central grey matter of midbrain at the level of
superior colliculi.
 Connected to the pyramidal tracts, pretectal nuclei (light reflex),
4th,6th& 8th nerve nuclei (coordination of eye movements).
 PARTS OF NUCLEAR COMPLEX:
1. Dorsolateral – to supply inferior rectus muscle.
2. Intermediate – to inferior oblique.
3. Ventromedial – to medial rectus.
4. Caudal central – to part of levator palpabrae superioris.
5. Median raphe – to superior rectus.
6. Edinger-Westphal – to ciliaris & sphincter pupillae muscles.

Page 40 of 284
CLINICAL ANATOMY
COMPLETE PARALYSIS OF NERVE:
 Ptosis. – dropping of upper eyelid.
 Lateral squint.
 Dilatation of pupil – paralysis of parasympathetic fibres to
sphincter pupillae muscle.
 Loss of accommodation – paralysis of ciliary muscles.
 Slight proptosis – forward projection of the eye.
 Diplopia or double vision.

COMPRESSION OF 3RD NERVE:


 Dilatation of pupil.
 Little response to light.

ANEURYSM OF POSTERIOR CEREBRAL SUPERIOR


CEREBELLAR ARTERY:
 Compression of 3rd nerve as it passes between them.

Page 41 of 284
MEDULLA OBLONGATA

INTRODUCTION:
 The medulla the lowest part of the brainstem.
 Direct upward continuation of the spinal cord.
 Extend- the upper border of pons to the foramen magnum
 Anteriorly, its related to the clivus and meninges.
 Posteriorly, to the vallecula of the cerebellum.
 Medulla provides attachment to the last four cranial nerves.
 The lower part of medulla, like that of spinal cord contains the
central canal.
 In the upper part of the medulla, this canal widens and moves
dorsally to form the lower part of the 4th ventricle. Thus, the
medulla has two parts:
1. Open/superior part- the dorsal surface of the medulla is formed
by the fourth ventricle.
2. Closed/ inferior part- fourth ventricle is narrowed at the apex
and continues with the central canal.

EXTERNAL FEATURES:
 The medulla is divided into right and left symmetrical halves by the
anterior median fissure and posterior median sulcus.
 The anterior median fissure is continuous below with the
corresponding fissure on the spinal cord, and above it ends into a
small triangular depression called the foramen cecum at the lower
border of the pons.
 The anterior median fissure is interrupted in its lower part by the
bundles of fibres crossing obliquely from one side to the other, the
decussation of pyramids.

Page 42 of 284
 The posterior median sulcus continues below with the corresponding
sulcus of the cord and is present only in the lower half of the
medulla. Above, its lips diverge to form the boundaries of a
triangular area, the lower part of the floor of the 4th ventricle.
 Each half of the medulla is marked by two sulci, anterolateral and
posterolateral, which are direct upward continuations of the
corresponding sulci of the cord.
 The anterolateral sulcus extends along the lateral border of the
pyramid and along it emerges the rootlets of the hypoglossal (12th
cranial) nerve.

Page 43 of 284
 The posterolateral sulcus lies between the olive and the inferior
cerebellar peduncle and along it emerges the rootlets of the
glossopharyngeal (9th), vagus (10th), and cranial root of
accessory (11th) cranial nerves.

Page 44 of 284
FEATURES ON THE VENTRAL ASPECT-
1. Pyramids: These are two elongated elevations, one on either side of
the anterior median fissure, and are produced by the underlying
corticospinal (pyramidal) fibres.
2. Olives: These are oval elevations, posterolateral to the pyramids and
are produced by an underlying mass of grey matter called inferior
olivary nucleus.
3. Rootlets of the hypoglossal nerve: These emerge from the
anterolateral sulcus.
4. Rootlets of the 9th, 10th, and 11th (cranial part) cranial
nerves: These emerge through the posterolateral sulcus

FEATURES ON THE DORSAL ASPECT-


The dorsal aspect of the medulla is well demarcated into lower closed
and upper open parts.
FEATURES OF THE CLOSED PART

1. On either side of the posterior median sulcus, presents three


longitudinal elevations. From medial to lateral these are: fasciculus
Gracilis, fasciculus cuneatus, and inferior cerebellar
peduncle.
2. The upper ends of the fasciculus Gracilis and fasciculus cuneatus
expand to form the gracile and cuneate tubercles, respectively, due to
underlying nuclei of gracile and cuneate nucleus.
3. Another elevation present lateral to cuneate tubercle i.e., the tuber
cinereum is produced by the spinal nucleus of the trigeminal nerve.

Page 45 of 284
FEATURES OF THE OPEN PART

1. The open part of the medulla forms the lower part of the floor of the
4th ventricle which presents a number of features such as, median
sulcus, hypoglossal and vagal triangles, vestibular areas, area
postrema, stria medullaris, etc.

INTERNAL STRUCTURE:
 The internal structure of medulla is well appreciated by examining
its transverse sections (T.S.) at the following three levels:
1. At the level of decussation of pyramids.
2. At the level of sensory decussation.
3. At the level of the olives.

T.S. OF MEDULLA AT THE LEVEL OF SENSORY


DECUSSATION-
GREY MATTER:
1. Lateral to the cuneate nucleus, the accessory cuneate nucleus lies
which relays unconscious proprioceptive fibres from the upper limbs.
Its equivalent to nucleus dorsalis / Clarke’s column.
2. The nucleus of the spinal tract of the trigeminal nerve is also
separated from the central grey matter.
3. The lower part the inferior olivary nucleus is seen.
4. The central grey matter contains the following:
a. HYPOGLOSSAL NUCLEUS- elongated nucleus about 2cm
long. It supplies the muscles of the tongue expect the
palatoglossus.

Page 46 of 284
b. DORSAL NUCLEUS OF THE VAGUS - gives preganglionic
parasympathetic fibres to heart and to smooth muscles and
glands of respiratory and alimentary systems.
c. NUCLEUS OF TRACTUS SOLITARIES- receives taste fibres.

WHITE FIBRES:
a. The nucleus Gracilis and cuneatus give rise to the internal
arcuate fibres. These fibers cross to the opposite side where
they form a paramedian band of fibres, called the medial
lemniscus.
b. The pyramid tracts lie anteriorly
c. The medial longitudinal bundle lies posterior to the medial
lemniscus.
d. The spinocerebrallar, lateral spinothalamic and other tracts lie
in the anterolateral area.
e. Emerging fibres of 12th nerve.

Page 47 of 284
BLOOD SUPPLY:
a. Anterior spinal artery- supplies medial part
b. Posterior inferior cerebellar artery- supplies posterolateral part.

CLINICAL ANATOMY:
LATERAL MEDULLARY (POSTERIOR INFERIOR
CEREBELLARARTERY) SYNDROME OF WALLENBERG:

 It occurs due to thrombosis the of posterior inferior cerebellar


artery. features are:
a. Contralateral loss of pain and temperature sensation in the
trunk and limbs, due to involvement of spinothalamic tract.
b. Ipsilateral loss of pain and temperature sensation over the
face, due to involvement of the spinal nucleus and tract of
the trigeminal nerve.
c. Ipsilateral paralysis of muscles of palate, pharynx, and
larynx, due to involvement of nucleus ambiguous.
d. Ipsilateral ataxia, due to involvement of inferior cerebral- lar
peduncle and cerebellum.
e. Giddiness, due to involvement of vestibular nuclei.

MEDIAL MEDULLARY SYNDROME:


 It occurs due to blockage of anterior spinal artery. Features are:
a. Contralateral hemiplegia/paralysis of arm and leg, due to
damage of pyramid.
b. Ipsilateral paralysis and atrophy of the half of the tongue,
due to damage of hypoglossal nerve.
c. Contralateral loss of position and vibration sense due to
damage of medial lemniscus.

Page 48 of 284
INJURY TO THE LOWER PART OF MEDULLA:

 Injury in this part may be fatal due to the involvement of the vital
centres (respiratory centres, vasomotor centres)
a. BULBAR PALSY:
 Weakness of muscles supplied by 7-12 cranial nerves.
The lesion is of lower motor neuron type. The patient has
difficult in speaking, swallowing and in usage of muscles
of facial expression.
b. PSEUDOBULBAR PALSY:
 Due to bilateral lesion of corticonuclear fibers. There is
paralysis of tongue, pharynx and palate.

Page 49 of 284
CEREBELLUM

INTRODUCTION:
 Cerebellum is the largest part of hindbrain.
 It maintains equilibrium, muscle tone and co-ordination.
 It lies in posterior cranial Fossa behind pons and medulla.
 It is separated from cerebrum by tentorium cerebelli.

EXTERNAL FEATURES:
 Cerebellum has two laterally expanded hemispheres connected
together in midline by narrow constricted part called vermis.
 It has two surfaces:
1. SUPERIOR SURFACE of the hemisphere and vermis lie in
one plane. This part of vermis is called superior vermis.
2. INFERIOR SURFACE has depression called Vallecula of
Cerebellum, for inferior vermis.
 Anterior aspect has a notch in which the pons and medulla are
lodged.

PARTS OF CEREBELLUM:
ANATOMICAL SUBDIVISIONS:
 The surface of the cerebellum (both the vermis and hemispheres)
is subdivided into numerous small leaves like bands by a series of
fissures that runs parallel to each other.
 Some fissures are deeper and divides the cerebellum into lobes.
 Subdivision of vermis according to Larsell's classification and
corresponding subdivisions of the cerebellar hemispheres.

Page 50 of 284
Page 51 of 284
MORPHOLOGICAL SUBDIVISIONS:
 Based on phylogenetic and functional criteria, it is divided into the
following:
ARCHICEREBELLUM (VESTIBULO-CEREBELLUM):
 Oldest part of cerebellum.
 It is made up of:
a. Flocculo- Nodular lobe
b. Lingula
 It is vestibular in its connections.
 Controls Axial musculature and the bilateral movements used for
locomotion and maintenance of equilibrium.
PALEOCEREBELLUM (SPINO-CEREBELLUM):
 It is made up of
a. Anterior lobe except Lingula.
b. Pyramid and uvula
 They have spinocerebellar connections and receive impulses from
spinal cord through spinocerebellar and reticulocerebellar
tracts.
 It is responsible for maintenance of posture and performance of
voluntary movements.
NEOCEREBELLUM:
 It is made up of posterior lobe except pyramid and uvula.
 They have cortical connections
 It regulates fine co-ordinates voluntary movements.

Page 52 of 284
FUNCTIONAL DIVISIONS OF CEREBELLUM:

INTERNAL STRUCTURE:
WHITE MATTER- central core of white matter arranged in the form of
branching pattern of tree known as Arbor Vitae cerebelli.
GREY MATTER- contains cerebellar cortex and cerebellar nuclei like
a. Nucleus Dentate
b. Nucleus Globous
c. Nucleus Emboliformis
d. Nucleus Fastigial.

Page 53 of 284
BLOOD SUPPLY:
a. Superior surface- Two superior cerebellar arteries – Branch of
Basilar artery
b. Anterior part of inferior surface – Two anterior inferior cerebellar
arteries – Branch of Basilar artery
c. Posterior part of inferior surface – Two posterior inferior
cerebellar arteries – Branch of vertebral artery.
d. Vein drains into neighbouring venous sinuses.

CONNECTIONS OF CEREBELLUM:
 Fibres entering and leaving cerebellum are grouped to form three
cerebellar peduncles.
SUPERIOR CEREBELLAR PEDUNCLES:
 Connects the midbrain with cerebellum
 Consists of fibres arising from cerebellar nuclei
 These fibres pass upwards, forwards and medially.
 Superior Medullary velum connects right and left superior
cerebellar peduncles forming roof of fourth ventricle.
 Fibres enter the tegmentum and cross to opposite side in
tegmental decussation.
 Many of the fibres end in red nucleus and some extend to
thalamus.

Page 54 of 284
 It is chiefly efferent in nature.
a. AFFERENTS:
 Anterior spinocerebellar
 Tectocerebellar
b. EFFERENTS:
 Globorubral
 Dentatothalamic
 Dentato- olivary
 Fastigio- Reticular

MIDDLE CEREBELLAR PEDUNCLE:


 Chiefly different to cerebellum
 Connects Basilar part of pons with cerebellum
 Begins as direct lateral continuation of Basilar parts of pons.
 Demarcation between the Basilar part of pons and middle
cerebellar peduncle is made by attachment of trigeminal nerve.
 It has Ponto-cerebellar fibres arising from pontine nuclei.

INFERIOR CEREBELLAR PEDUNCLE:


 It is also chiefly afferent to cerebellum
 It is also called as restiform body.
 Connects posterolateral part of medulla with cerebellum.
 Peduncles passes upwards and laterally along the inferolateral
margin of the floor of fourth ventricle.
 Near the upper end of medulla, it lies between superior and
middle cerebellar peduncles.
 The inferior cerebellar peduncle makes a sharp bend to enter the
white matter of cerebellum, at this site, vestibulo-cochlear nerve is
attached to the peduncle.
 Fibres arising from vestibular nuclei pass over the medial part of
peduncle and these fibres constitute the juxta restiform body.
Page 55 of 284
AFFERENTS:
 Posterior spinocerebellar fibres
 Cuneo-cerebellar fibres (posterior external arcuate fibres).

FUNCTIONS:
 It controls same side of the body
 Its co-ordinates voluntary moments so that they are smooth,
balanced and accurate.
 Tone, posture and equilibrium is maintained by
archicerebellum and paleocerebellum.
 It controls movements of eyeball.
 It plays role in cognition.

APPLIED ANATOMY:
1. CEREBELLUM COGNITIVE AFFECTIVE SYNDROME:
 Due to thrombosis of one of six arteries supplying cerebellum
 The patient shows inattention, grammatical errors in speech
and patchy memory loss.
 It is characterised by
a. Muscular hypotonia
b. Intention tremors
c. Adiadochokinesia
d. Nystagmus
e. Ataxia

MICROSCOPIC ANATOMY:
 Cerebellum has uniform structure, that is homotypical.
 Cortex contains three layers:

Page 56 of 284
1) MOLECULAR LAYER – Contains unmyelinated nerve
fibres derived from axons of granule cells, axons of stellate
cells and basket cells, sensory climbing fibres, dendrites of
purkinje and Golgi cells
2) INTERMEDIATE LAYER – Contains bodies of purkinje cells
3) INNER LAYER – made of cell bodies and dendrites of Golgi
cells

 There are five types of cells:


a. Purkinje cell
b. Granule cell
c. Stellate cell
d. Basket cell
e. Golgi cell
 Except granular cells all are inhibitory in nature.
a. MOSSY FIBRES – All afferent except olivocerebellar fibres,
release glutamate
b. CLIMBING FIBRES - Fibres from inferior olivary nucleus,
release Aspartate.

Page 57 of 284
HYPOTHALAMUS

INTRODUCTION
 Part of diencephalon, which lies below the thalamus.
 Forms floor and lateral wall of third ventricle.

BOUNDARIES
1. ANTERIOR - lamina terminalis
2. POSTERIOR -subthalamus
3. INFERIOR- floor of third ventricle
4. SUPERIOR -thalamus
5. LATERAL- internal capsule
6. MEDIAL - cavity of 3rd ventricle

REGION AND NUCELI


a. HYPOTHALAMUS- subdivided anteroposteriorly into 4 region,
preoptic region
b. Supraoptic region

REGION NUCLUES

Preoptic region Preoptic nucleus

Supraoptic nucleus
Supraoptic region Anterior nucleus
Paraventricular nucleus

Arcuate nucleus
Tuberal region
Ventromedial nucleus
Page 58 of 284
Dorsomedial nucleus

Mammillary region Posterior nucleus, mammillary nucleus

c. Tuberal region - (infundibulum, tuber cinereum)

RELATIONS:
 The hypothalamus has an almost geometrically central location
in the brain.
 Immediately anteriorly, there is the anterior commissure and
the lamina terminalis. The subcallosal area with its gyrus is
located anteriorly, just beyond the anterior commissure and
lamina terminalis.
 Posteriorly, the mammillary bodies, posterior perforated
substance, cerebral peduncle and the cerebral aqueduct of
Sylvius can be found.
 Inferiorly (from anterior to posterior), there is the supraoptic
recess and crest, the pituitary stalk and gland, and the tuber
cinereum.
 Superiorly there is the hypothalamic sulcus, the thalamus,
the choroid plexus of the third ventricle and the fornix.

BLOOD SUPPLY:

a. Anterior and posterior branches of circle of Willis


b. Superior hypophyseal artery

FUNCTIONS -HYPOTHALAMUS
a. Autonomic control
b. Endocrine control
c. Neurosecretion

Page 59 of 284
d. Regulation of food and water intake
e. Emotional expression
f. Sexual behaviour and reproduction
g. Temperature regulation
h. Biological clock

 Cardiovascular regulation - lateral and posterior hypothalamus


increases arterial pressure, preoptic area - decreases arterial
pressure.
 Body temperature- anterior hypothalamus - heat loss
 Posterior hypothalamus-heat gain
 Body water regulation - supraoptic area paraventricular area-
Secrete antidiuretic hormone- increases reabsorption of water.
 Uterine contractility and milk ejection - paraventricular nucleus
(oxytocin)
 Hunger - lateral hypothalamus, ventromedial nucleus – satiety
 Appetite - arcuate nuclei
 Mammillary bodies - control of GI, reflex of feeding
 Circadian rhythm - suprachiasmatic nucleus
 Reward centre – Lateral and ventromedial nuclei
 Punishment centre – medial hypothalamus

NEUROENDOCRINE PATHWAYS:
 Neurons of the supraoptic and the paraventricular nucleus in the
hypothalamus project to the neurohypophysis forming
hypothalamo-neurohypophyseal neurosecretory system.
a. Fibers of these neurons compose the hypothalamo-hypophyseal
tract.
b. These neuronal fibres secrete oxytocin and ADH (vasopressin)
 These hormones are delivered to the pituitary by hypothalamo-
hypophyseal portal system and are stored in the pituitary.

Page 60 of 284
APPLIED ANATOMY
Lesions of hypothalamus result in
a. Frohlich syndrome: Hypothalamic obesity and hypogonadism.
b. Diabetes insipidus due to diminished secretion of ADH. Polyuria
is seen.
c. Diencephalic autonomic epilepsy
d. Sleep disturbances like narcolepsy and cataplexy
e. Sexual disturbances

Page 61 of 284
3RD VENTRICLE

GROSS ANATOMY
 It is a slit like cavity and is present in the diencephalon in the
midline
 It is located in between the 2 thalami.
 It extends from lamina terminalis anteriorly and superior end of
the cerebral aqueduct posteriorly
 It is lined by ciliated columnar epithelium along with the
ependyma.
 The interthalamic adhesion traverses the third ventricle
horizontally.

Page 62 of 284
 It communicates with the lateral ventricle anteriorly via the
interventricular foramen of Monroe and posteriorly with the 4th
ventricle via the cerebral aqueduct of Sylvius.

BOUNDARIES
ROOF: Ependyma which extends over the 2 thalami
ANTERIOR WALL:
a. Lamina terminalis
b. Anterior commissure
c. Anterior column of fornix
POSTERIOR WALL:
a. Pineal gland
b. Posterior commissure
c. Opening of Cerebral aqueduct
FLOOR:
a. Optic chiasma
b. Tuber cinereum and infundibulum
c. Mammillary bodies
d. Posterior perforated substance
e. Tegmentum of midbrain
LATERAL WALL:
 It is divided into Upper part and Lower part by the
hypothalamic sulcus extending from interventricular foramen to
the upper end of cerebral aqueduct.
a. Upper part: Medial surface of the anterior 2/3rd of thalamus
b. Lower Part: Hypothalamus

Page 63 of 284
RECESSES
The 3rd ventricle has 5 recesses:
1. INFUNDIBULAR RECESS: It extends into the Sella turcica as
the stalk of the pituitary gland.
2. OPTIC RECESS: It’s an angular recess situated at the junction
of the anterior wall and the floor of the ventricle just above the
optic chiasma
3. ANTERIOR RECESS: It’s a triangular recess extending in front
of the interventricular foramen of Monroe and behind the anterior
commissure
4. SUPRAPINEAL RECESS: It extends posteriorly above the stalk
of the pineal gland and below the Tela choroidea.
5. PINEAL RECESS: It extends posteriorly between the superior
and inferior lamina of the stalk of the pineal gland.

APPLIED ANATOMY
OBSTRUCTION OF THE 3RD VENTRICLE: the 3rd ventricle is
easily blocked by local brain tumors or congenital defects. This causes
the accumulation of CSF causing an increased intracranial
pressure in adults and hydrocephalus in children.

Page 64 of 284
SUPEROLATERAL SURFACE OF CEREBRAL
HEMISPHERE
 The
cerebrum is the largest part of the human brain that fills the most of
the cranial cavity.
 Its large
size is the result of a progressive centralization (telencephalization)
of the various higher sensory and motor centres of the brain during
evolution.
SUPEROLATERAL SURFACE OF THE CEREBRAL HEMISPHERE:
 The superolateral surface is most convex and most extensive. It
faces upwards and laterally and conforms to the corresponding half
of the cranial vault.

Sulci and Gyri on the superolateral surface of the cerebral hemisphere:

IN THE FRONTAL LOBE

Page 65 of 284
 The prefrontal sulcus often broken into two or three parts, runs
downwards and forwards parallel and little anterior to the central
sulcus. The area between the central and precentral sulci is called
precentral gyrus.
 Anterior to the precentral sulcus there are two sulci called superior
and inferior frontal sulci which run horizontally. These sulci divide
the region of frontal lobe in front of precentral sulcus into superior,
middle, and inferior frontal gyri.
 The anterior and ascending rami of lateral sulcus divide the inferior
frontal gyrus into three parts. The part below the anterior ramus is
called pars orbitalis, the part between the anterior and ascending
rami the pars trian-gularis and the part posterior to the ascending
ramus, the pars opercularis.

IN THE PARIETAL LOBE


 The postcentral sulcus runs downwards and forwards, a little behind
and parallel to the central sulcus.
 The area between these two sulci is called the postcentral gyrus.
 The rest of the parietal lobe is divided into a superior and inferior
parietal lobule by an intraparietal sulcus which runs horizontally
backwards from the postcentral sulcus.
 The upturned posterior end of the posterior ramus of lateral sulcus,
and the posterior ends of superior and inferior temporal sulci extends
into the inferior parietal lobule to divide it into three parts: (a) the
part that surrounds the posterior ramus of lateral sulcus is called
supra marginal gyrus, (b) the part surrounding the superior
temporal sulcus, the angular gyrus, and (c) the part surrounding the
inferior temporal sulcus, the arcus temporo-occipitalis.

IN THE TEMPORAL LOBE


 There are two sulci in this lobe that run parallel to the posterior
ramus of the lateral sulcus. These are termed superior and

Page 66 of 284
inferior temporal sulci, and divide the temporal lobe into superior,
middle and inferior temporal gyri.
 The superior surface of superior temporal gyrus presents two
transverse temporal gyri. The anterior transverse temporal gyrus
also called Heschl's gyrus forms the primary auditory area of the
cortex.

IN THE OCCIPITAL LOBE


 The occipital lobe possesses rather three short sulci, lateral and
transverse occipital sulci and lunate sulcus.
a. Lateral occipital sulcus runs horizontally and divides this
lobe into superior and inferior occipital gyri.
b. Lunate sulcus is C-shaped sulcus with forward convexity
just in front of the occipital pole.
c. Transverse occipital sulcus runs downwards into the
uppermost part of the occipital lobe from the supero-medial
border of hemisphere, a little behind the parieto-occipital
sulcus.

Functional areas of the cerebral cortex in the superolateral surface of


the cerebral hemisphere:

Page 67 of 284
IN FRONTAL LOBE:

PRIMARY MOTOR AREA (AREA 4 OF BRODMANN)


 The primary motor area is located in the precentral gyrus on the
superolateral surface of the hemisphere and in the anterior part of
the paracentral lobule on the medial surface.
 When these areas are stimulated electrically, contractions occur in
muscles of opposite half of the body.
 Anatomically, these areas give origin to projection fibres that form
the corticospinal and corticonuclear tracts.

PREMOTOR AREA (AREAS 6 AND 8 OF BRODMANN)


 The premotor area is located just anterior to the motor area. It
occupies the posterior parts of the superior, middle, and inferior
frontal gyri.
 Stimulation of the premotor area results in movements, but these are
more intricate than those produced by stimulation of the motor area.
 The premotor area is responsible for programming the intended
movements and control of movements in progress.

Page 68 of 284
FRONTAL EYE FIELD (AREA 8 OF BRODMANN)
 The frontal eye field lies in the middle frontal gyrus.
 Stimulation of this area causes both eyes to move to the opposite
side. These are called conjugate movements.

MOTOR SPEECH AREA (AREAS 44 AND 45 OF BRODMANN)


 The motor speech area of Broca lies in the inferior frontal gyrus of
the dominant hemisphere. Injury to this region results in inability to
speak (aphasia).
 These effects occur only if damage occurs in the left hemisphere in
right-handed persons and also in many left-handed persons.

PREFRONTAL AREA
 The part of frontal lobe rostral to motor and premotor areas.
 It determines the initiative and judgement of an individual, also
concerned with depth of emotions, social, moral and ethics.
IN PARIETAL LOBE:

PRIMARY SOMATOSENSORY AREA (AREAS 3, 1 & 2 OF


BRODMANN)
 The sensory area located in the postcentral gyrus is called the
primary somatosensory area.
 It also extends onto the medial surface of the hemisphere where it
lies in the posterior part of the paracentral lobule.
 It receives projections from the ventral posteromedial and ventral
posterolateral nuclei of the thalamus.
 The area of cortex that receives sensations from a particular part of
the body is not proportional to the size of that part but rather to the
complexity of sensations received from it.
 Thus, the digits, the lips, and the tongue have disproportionately
large representation.

Page 69 of 284
SECONDARY SENSORY AREA
 It is located in upper lip of posterior ramus of lateral sulcus. It
related to pain perception.

SOMATOSENSORY ASSOCIATION AREA (AREAS 5 AND 7 OF


BRODMANN)
 Parts of the superior parietal lobule help us recognize shape, size,
and texture of objects.

SENSORY SPEECH AREA (AREAS 22, 39 AND 40 OF


BRODMANN)
 The sensory speech area of Wernicke lies in the posterior part of the
superior temporal gyri of the left dominant hemisphere.
 It extends into areas 39 and 40 of the parietal lobe.
 This area is responsible for interpretation of speech.

IN TEMPORAL LOBE:

AUDITORY (ACOUSTIC) AREAS (AREAS 41, 42 AND 22 OF


BRODMANN)
 The primary auditory area or the area for hearing is situated in the
temporal lobe. It lies in that part of the superior temporal gyrus,
which forms the inferior wall of the posterior ramus of the lateral
sulcus.
 In this location, there are two short oblique gyri called the anterior
and posterior transverse temporal gyri.
 The auditory area lies in the anterior transverse temporal gyrus and
extends to a small extent onto the surface of the hemisphere in the
superior temporal gyrus (area 41, 42).

Page 70 of 284
 It receives the auditory radiation.
 Auditory association area lies in the superior temporal gyrus
(Brodmann’s area 22) posterior to primary auditory area.
 Hence, the auditory areas in each cerebral cortex receive fibres from
both the right and left cochleae.

IN OCCIPITAL LOBE:

VISUAL ASSOCIATION AREA (AREA 18)


 It surrounds the primary visual area and occupies visual cortex of
medial and superolateral surface of cerebral hemispheres.

ARTERIAL SUPPLY:

 The functional areas in the frontal lobe of superolateral surface of


the cerebral hemisphere is supplied by middle cerebral and anterior
cerebral artery.
 The functional areas in the parietal lobe of superolateral surface of
the cerebral hemisphere is supplied by middle cerebral and anterior
cerebral artery.
 The functional areas in the temporal lobe of superolateral surface of
the cerebral hemisphere is supplied by middle cerebral and posterior
cerebral artery.
 The functional areas in the occipital lobe of superolateral surface of
the cerebral hemisphere is supplied by posterior cerebral artery.

Page 71 of 284
ARTERIAL SUPPLY OF CEREBRAL
HEMISPHERE:

ARTERIAL SUPPLY OF SUPEROLATERAL SURFACE


1. MIDDLE CEREBRAL ARTERY:
 About 2/3rd of superolateral surface is supplied by the middle
cerebral artery.
 The region of cerebral cortex supplied by it includes the greater
parts of primary motor and sensory areas, and frontal eye field.
 In the left (dominant) hemisphere it includes the Broca and
Wernicke’s speech areas.

2. ANTERIOR CEREBRAL ARTERY:


 A narrow strip of the cerebral cortex (about 2.5 cm in width)
adjoining superomedial border up to the parieto-occipital sulcus is
supplied by anterior cerebral artery.
Page 72 of 284
 The upper parts of primary motor and sensory areas lie in this
region.

3. POSTERIOR CEREBRAL ARTERY:


 A narrow strip along the lower border of temporal lobe (excluding
temporal pole) and occipital lobe are supplied by posterior cerebral
artery.
 The posterior part of visual area fall lies in this region.

ARTERIAL SUPPLY OF MEDIAL SURFACE


 The medial surface of the cerebral hemisphere is supplied by the
following arteries:
1. ANTERIOR CEREBRAL ARTERY:
 Most of the medial surface (anterior 2/3rd) is supplied by the
anterior cerebral artery.
 The region of cerebral cortex supplied includes the parts of motor
and sensory areas (paracentral lobule) concerned with perineum,
leg, and foot.

2. MIDDLE CEREBRAL ARTERY:


 Temporal pole of the temporal lobe is supplied by the middle
cerebral artery.

3. POSTERIOR CEREBRAL ARTERY:


 Occipital lobe is supplied by the posterior cerebral artery. The
area supplied includes the visual cortex.

Page 73 of 284
ARTERIAL SUPPLY OF INFERIOR SURFACE:
 The inner surface of the cerebral hemisphere is supplied by the
following arteries:

1. POSTERIOR CEREBRAL ARTERY:


 Most of the inferior surface except the temporal pole is supplied by
the posterior cerebral artery.

2. MIDDLE CEREBRAL ARTERY:


 Lateral part of the orbital surface of the frontal lobe and temporal
pole of the temporal lobe are supplied by the middle cerebral
artery.

3. ANTERIOR CEREBRAL ARTERY:


 Medial part of the orbital surface of the frontal lobe is supplied by
the anterior cerebral artery.

CLINICAL CORRELATION
OCCLUSION OF ANTERIOR CEREBRAL ARTERY:
 The occlusion distal to the anterior communicating artery
produces the following symptoms:
a. Contralateral hemiparesis and hemianesthesia involving leg
and foot, due to involvement of upper parts of primary motor
and sensory areas, and parental lobules.

Page 74 of 284
b. Inability to identify the objects correctly, involvement of
superior parietal globules.
c. Apathy and personality changes, involvement of part of
frontal lobe.

OCCLUSION OF MIDDLE CEREBRAL ARTERY:


 The occlusion of middle cerebral artery occurs commonly. It
produces the following signs and symptoms:
a. Contralateral hemiplegia and hemianesthesia involving
mainly the face and arm, due to involvement of most of the
primary motor and sensory areas.
b. Aphasia, if left dominant hemisphere is involved—due to
involvement of motor and sensory speech areas.
c. Contralateral homonyms hemianopia, due to involvement of
optic radiation

OCCLUSION OF POSTERIOR CEREBRAL ARTERY:


 Contralateral homonyms hemianopia, due to involvement of visual
cortex with some degree of macular sparing. The macular vision is
spared.

Page 75 of 284
WHITE MATTER OF CEREBRUM

WHITE MATTER:
 It consists chiefly of myelinated fibres which connect various parts
of cortex to one another & also to other parts of the CNS.

 Three types of white fibres are:


a. Association fibres.
b. Commissural fibres.
c. Projection fibres.

ASSOCIATION FIBRES
 The association fibres interconnect the different regions of the
cerebral cortex in the same hemisphere. the two types:
a. SHORT ASSOCIATION FIBRES (U fibres) which
interconnect the adjacent gyri by hooking around the sulcus,
hence they are also called arcuate fibres.
b. LONG ASSOCIATION FIBRES, which travel for long
distances and interconnect the widely separated gyri.

Page 76 of 284
EXAMPLES:
1. UNCINATE FASCICULUS – connects temporal pole to motor
speech area & to orbital cortex.
2. CINGULUM - connects cingulate gyrus to para-hippocampal
gyrus.
3. SUPERIOR LONGITUDINAL FASCICULUS – connects frontal to
occipital & temporal lobes. It is the longest association bundle.
4. INFERIOR LONGITUDINAL FASCICULUS – connects occipital
& temporal lobes.
5. FRONTO – OCCIPITAL FASCICULUS – connects frontal pole to
occipital & temporal lobes. Similar course to Superior longitudinal
fasciculus but it (from – occipital bundle) is deeply situated and
separated from the former by fibres of Corona radiata.

COMMISSURAL FIBRES
 The commissural fibres interconnect the identical cortical areas of
the two cerebral hemispheres. The bundles of such fibres are
termed commissures.
 Importance: These fibres are essential for interhemispheric
transfer of information for bilateral responses & learning
processes.
 The important commissures of the brain are as follows:
1. CORPUS CALLOSUM – connects cerebral cortex of two sides.
2. ANTERIOR COMMISSURE – connects archipallium (olfactory
bulbs, piriform area, anterior parts of temporal lobes) of two
sides.
3. POSTERIOR COMMISSURE – it interconnects the superior
colliculi, pretectal & interstitial nuclei of two sides.
4. HIPPOCAMPAL COMMISSURE (commissure of fornix) –
connects crura of fornix & thus hippocampal formations of two
sides.
5. HABENULAR COMMISSURE – connects habenular nuclei
Page 77 of 284
6. Corpus callosum is the largest commissure of the brain.

PROJECTION FIBRES
 The projection fibres connect the cerebral cortex to the subcortical
centres (corpus striatum, thalamus, brainstem) and spinal cord.
 They are of two types:
i. Corticofugal fibres, which go away from the cortex to
centres in the other parts of the CNS.
ii. Corticotectal fibres, which come to the cerebral cortex
from the other centres in the CNS.
 The most important projection fibres are internal capsule &
fornix.

INTERNAL CAPSULE
 The internal capsule is a compact bundle of projection fibres between
1. MEDIALLY: thalamus and caudate nucleus
2. LATERALLY: lentiform nucleus laterally.
 The fibres fan out rostrally to form Corona radiata & condense
caudally to continue as crus cerebri of midbrain.
 It consists of ascending and descending nerve fibres which connect
the cerebral cortex to the brainstem and spinal cord.

PARTS AND RELATIONS


 It is a V Shaped compact bundle of the white fibres which is concave
laterally.
 The internal capsule is divided into the following five parts:
1. ANTERIOR LIMB—between the head of caudate nucleus and
the anterior part of the lentiform nucleus.

Page 78 of 284
2. POSTERIOR LIMB—between the thalamus and the posterior
part of the lentiform nucleus.
3. GENU—the bend between the anterior and posterior limbs
with concavity of the bend facing laterally.
4. RETROLENTIFORM PART—behind the lentiform nucleus.
5. SUBLENTIFORM PART—below the lentiform nucleus.

CONSTITUENT FIBRES OF INTERNAL CAPSULE &


ARTERIAL SUPPLY
 It consists of motor (descending) fibres and sensory (ascending)
fibres.

Page 79 of 284
CLINICAL ANATOMY:
1. VASCULAR LESIONS ON MEDIAL & LATERAL STRIATE
BRANCHES OF MIDDLE CEREBRAL ARTERY – Results in
hemiplegia of opposite half of the body including face – upper
motor neuron paralysis.
2. LARGER STRIATE ARTERY – known as Charcot's artery of
cerebral haemorrhage.
3. THROMBOSIS OF RECURRENT BRANCH OF ANTERIOR
CEREBRAL ARTERY – upper motor neuron paralysis of opposite
upper limb & face
4. LESION ON GENU – sensory & motor loss in C/L side of head but
partial loss due to bilateral cortical innervation of most cranial
nerve nuclei.

Page 80 of 284
YPOTHALAMUS
 Hypothalamus is a part of diencephalon, lies below the thalamus. It
forms the floor and the lower parts of lateral walls of the 3rd
ventricle.
 Hypothalamus controls the autonomic and endocrine activities of the
body majorly and emotional behaviour.
 Being the principal autonomic centre of the brain, it has been
regarded as the head ganglion of the autonomic nervous system by
Sherrington.

BOUNDARIES OF THE HYPOTHALAMUS:


1. ANTERIORLY: Lamina terminalis (lamina terminalis extends
from the optic chiasma to the anterior commissure).
2. POSTERIORLY: Subthalamus.
3. INFERIORLY: Structures in the floor of the 3rd ventricle - tuber
cinereum, infundibulum and mammillary bodies. (These
structures are actually the parts of hypothalamus itself.)
4. SUPERIORLY: Thalamus.
5. LATERALLY: Internal capsule.
6. MEDIALLY: Cavity of the 3rd ventricle.

SUBDIVISIONS OF THE HYPOTHALAMUS:


 The hypothalamus is subdivided anteroposteriorly into the
following four regions:
a. Preoptic region—adjoining the lamina terminalis.
b. Supraoptic region—above the optic chiasma.
c. Tuberal region—includes the tuber cinereum, infundibulum
and area around it.
d. Mammillary region—includes the mammillary bodies and
area around it.

Page 81 of 284
 The Hypothalamus is divided from medial to lateral side into
following three zones.
1. Periventricular zone – part of medial zone
2. Intermediate zone – part of medial zone
3. Lateral zone – part of lateral zone

HYPOTHALAMIC NUCLEI
 Hypothalamus consists of numerous cell groups called
hypothalamic nuclei.
 Hypothalamic regions and nuclei in them.

Regions Nucleus/nuclei

Preoptic region Preoptic nucleus

Supraoptic nucleus
Supraoptic region Anterior nucleus
Paraventricular nucleus

Arcuate nucleus
Tuberal region Ventromedial nucleus
Dorsomedial nucleus

Posterior nucleus
Mammillary region
Mammillary nucleus

Page 82 of 284
FUNCTIONS OF HYPOTHALAMIC NUCLEI:

Page 83 of 284
IMPORTANT CONNECTIONS:

Page 84 of 284
CLINICAL ANATOMY:
 Lesions of hypothalamus gives rise to one of the following
syndromes:
1. Obesity - Frolich's syndrome
2. DI
3. Diencephalic autonomic epilepsy – characterised by flushing,
sweating, salivation, lacrimation, tachycardia, retardation of
respiratory rate, unconsciousness
4. Sexual disturbance – precocity / impotence
5. Disturbance of sleep – somnolence / narcolepsy
6. Hyperglycaemia & glycosuria

Page 85 of 284
NUCLEI OF THALAMUS AND THEIR
CONNECTIONS

INTRODUCTION:
 Internally a
“Y” shaped bundle of white matter called internal medullary
lamina divides the grey matter of thalamus into three major

groups of nuclei: Anterior, medial and lateral.

Page 86 of 284
SUBDIVISIONS OF THALAMIC NUCLEI

OTHER THALAMIC NUCLEI: -


1. Intralaminar
nuclei
2. Embedded
within internal medullary lamina
3. There are
several nuclei in this group: most important-
4. Centromedian
nucleus.
5. Medial nuclei
6. Medial and
lateral geniculate bodies

Page 87 of 284
FUNCTIONAL CLASSIFICATION OF THALAMIC NUCLEI: -

• V
MOTOR
entral anterior nucleus
RELAY
• V
GROUP
entral lateral nucleus
• V
entral posterolateral nucleus
• V
SENSORY
entral posteromedial nucleus
RELAY
• M
GROUP
edial geniculate body
• L
ateral geniculate body
• L
ateral dorsal nucleus
SENSORY
• L
MODULAT
ateral posterior nucleus
OR GROUP
• P
ulvinar
LIMBIC • A
GROUP nterior nucleus

Page 88 of 284
• M
edial dorsal nucleus
• I
NONSPECI ntralaminar nuclei
FIC • M
THALAMIC idline nuclei
NUCLEI • R
eticular nuclei

CONNECTIONS
 Connections of
anterior group of thalamic nuclei.

 Connections of
medial dorsal nucleus of thalamus

Page 89 of 284
CONNECTIONS OF LATERAL GROUP OF THALAMIC
NUCLEI

Page 90 of 284
CONNECTIONS OF INTRA-LAMINAR THALAMIC
NUCLEI

CONNECTIONS OF MEDIAL GENICULATE BODY

Page 91 of 284
CONNECTIONS OF LATERAL GENICULATE BODY

CONNECTIONS OF MIDLINE NUCLEI OF THALAMUS

Page 92 of 284
Page 93 of 284
FUNCTIONS: -

NAME FUNCTIONS
Relay station for hippocampal impulses
Anterior nucleus
for emotions and recent memory.
Relay station for visceral impulses,
integration of visceral somatic, olfactory
Medial nucleus
impulses, integration of visceral somatic,
olfactory impulses, related to emotions.
Lateral nuclei: Lateral
dorsal, Lateral posterior and Correlative function
pulvinar.
Relay station for striatal impulses, activity of
Ventral anterior nucleus
motor cortex influenced.

Page 94 of 284
Relay station for cerebellar impulses, activity
Ventral lateral nucleus
of motor cortex influenced.
Relay station for exteroceptive and
Ventral posterolateral
proprioceptive impulses from body, to
nucleus
consciousness.
Ventral posteromedial Relay station for impulses from the face, head
nucleus and taste impulses to consciousness.

Intralaminar, midline, and Participate in arousal reactions or ascending


reticular nuclei reticular activating system

Centromedian nucleus Receiving pain fibres


Relay station for auditory impulses and
Medial geniculate body
perception for painful and nociceptive stimuli.
Lateral geniculate body Relay station of visual impulses

CLINICAL ANATOMY-THALAMIC PAIN: -


 THALAMIC PAIN is a severe and treatment-resistant type of
central pain that may develop after thalamic stroke.
 Lesions within the ventrocaudal regions of the thalamus carry
the highest risk to develop pain, but its emergence in individual
patients remains impossible to predict.

Page 95 of 284
BASAL GANGLIA
INTRODUCTION
 Basal nuclei (basal ganglia) - Subcortical and intracerebral masses
of grey matter.
 Developed from telencephalon.

COMPONENTS
ANATOMICALLY
CORPUS STRIATUM - Partially divided by internal capsule into two
nuclei:
1. The caudate nucleus
2. The lentiform nucleus

Caudate nucleus + lentiform nucleus = corpus striatum

Caudate nucleus +putamen =neostriatum Globus pallidus-paleostriatum

3. The amygdaloid body Archistriatum


4. Claustrum

FUNCTIONALLY
 Corpus striatum, subthalamic nucleus, substantia nigra, ventral
striatum.

Page 96 of 284
SITUATION
1. The CAUDATE NUCLEUS “C”-shaped subcortical structure
which lies deep inside the brain near the thalamus.
2. The LENTIFORM NUCLEUS is a large, lens-shaped mass of
Gray matter just lateral to the internal capsule.
3. CLAUSTRUM is a thin lamina of grey matter that lies lateral to
the lentiform nucleus.
4. AMYGDALOID is in the temporal lobe of cerebral hemisphere
and close to temporal pole.

Page 97 of 284
RELATIONS AND CONNECTIONS: -
RELATIONS OF CAUDATE NUCLEUS: -

 Surrounded by lateral ventricle


 The concavity of ‘c’ encloses the thalamus and internal capsule.
 Head of the caudate nucleus forms:
1. Floor of anterior horn of lateral ventricle
2. Medial wall of anterior limb of internal capsule.
 Body forms floor of central part of lateral ventricle. Superiorly, it
is related to the fronto- occipital bundle and the corpus callosum.
 Tail forms roof of inferior horn of lateral ventricle, and ends by
joining the amygdaloid body at the temporal pole.
 It is medially related to stria terminalis, laterally to the tapetum,
and superiorly to the Sublentiform part of internal capsule and to
the globus pallidus.

RELATIONS OF LENTIFORM NUCLEUS: -

 Lateral surface is related to external capsule-the claustrum, the


outermost capsule, insula and is grooved by the lateral striate
arteries.
 Medial surface is related to the internal capsule- caudate nucleus
and thalamus.
 Inferior surface is related to Sublentiform part of internal capsule.

CONNECTIONS OF CORPUS STRIATUM: -

AFFERENT CONNECTIONS: -

 Entire cerebral cortex via corticostriate fibres.


 Intralaminar nuclei of thalamus via thalamostriate fibres.
 Pars compacta of substantia nigra via nigrostriatal fibres

Page 98 of 284
 Noradrenergic fibres received from locus coeruleus.
 Serotoninergic fibres received from raphe nucleus.

EFFERENT CONNECTIONS: -

1. Fasciculus lenticularis-arise from internal segment of globus


pallidus and enters the subthalamic region.
2. Ansa lenticularis-arise from both internal and outer segment of
globus pallidus and enters subthalamic region where it meets the
dentato-rubrothalamic fibres and the fasciculus lenticularis. The
union of the three tracts is called thalamic fasciculus, which
terminates in ventral anterior, ventral lateral and subthalamic
nuclei of thalamus.
3. Subthalamic fasciculus consists of reciprocal connections
between the globus pallidus and nucleus subthalamus.
4. Some fibres from globus pallidus also pass to substantia nigra
(pallidonigral fibres).

RELATIONS AND CONNECTIONS OF AMYGDALOID BODY: -


RELATIONS: -

 Functionally related to stria terminalis.


 Part of limbic system
 It lies deep to uncus and is related to anterior end of the inferior
horn of lateral ventricle.

CONNECTONS: -

 Afferents: From olfactory tract


 Efferents: It gives rise to stria terminalis which ends in anterior
commissure, the anterior perforated substance and in hypothalamic
nuclei.

Page 99 of 284
RELATIONS AND CONNECTIONS OF CLAUSTRUM: -
 It is separated from the latter fibres of the external capsule.
 Laterally, it is separated by thin layer of white matter from cortex of
insula.

Relations of caudate nucleus with the cavity of lateral ventriclePage 100 of 284
and thalamus
Relations of corpus striatum to internal capsule.

Connections of corpus striatum

Page 101 of 284


BLOOD SUPPLY: -
 Lenticulostriate branches of middle and anterior cerebral arteries.
 Anterior choroidal branch of internal carotid artery.

APPLIED ANATOMY: -
1. Sydenham’s chorea: Sometimes it occurs as complication of
rheumatic fever. The pathology is seen is striatum.
2. Huntington’s chorea: Autosomal dominant degenerative disease
of striatum and cerebral cortex.
3. Ballismus: Characterized by involuntary and violent movements
involving more proximal joints. Haemorrhagic involvement of
subthalamic nucleus causes violent, flinging movements
(hemiballismus) on the contralateral side of affected subthalamus.
4. Parkinsonism or paralysis agitans: It is also called as shaking
palsy. It is characterized by marked rigidity, which leads to
stooping posture, a slow- shuffling gait, difficulty in speech, and
mask like face. Characteristic “pill rolling” movements of hand are
seen. The condition is believed to be due to degenerative changes
in striatum and substantia nigra.
5. Wilson’s disease (Hepatolenticular degeneration): It is a
rare autosomal recessive inherited disorder of copper metabolism
resulting due to accumulation of copper in the liver. The pathology
is degeneration of lentiform nucleus. Accumulation of copper in
eyes is characterized by Kayser-Fleischer rings, which are whitish
rings at sclero-corneal junction or limbus.

Page 102 of 284


MENINGES AND MENINGEAL SPACES

DURAMATER has:
1. OUTER ENDOSTEAL LAYER - Attaches to endosteum
2. INNER MENINGEAL LAYER- surrounds the brain

A. OUTER LAYER (OR ENDOSTEAL LAYER)


 The outer layer is really the periosteum lining the inner surfaces of
the skull bones and is commonly referred to as endocranium.

Page 103 of 284


 It presents the following features:
1. It is continuous with the periosteum (pericranium) on the outer
surface of the skull through sutures and the foramina of the
skull.
2. It provides sheaths for cranial nerves, which fuse with
epineurium. The Dural sheath around the optic nerve fuses
with the sclera of the eyeball.
3. It is loosely attached to the inner surface of the vault of the
skull by numerous fibrous and vascular processes.
4. Meningeal vessels pass through this layer to supply the bone.

B. INNER LAYER (OR MENINGEAL LAYER)


 The inner layer is really the dura mater proper. It encloses the
brain and at the foramen magnum becomes continuous with
the dura mater surrounding the spinal cord (i.e., spinal dura).
It is separated from the arachnoid mater by a subdural space.
 At places, the meningeal layer is folded on itself to form Dural
folds (also called Dural septa).

ARACHNOID MATER
 The arachnoid mater is a transparent membrane, which invests
the brain
 loosely and continues as the spinal arachnoid at the foramen
magnum, which ends at the level of second sacral vertebra.
 It is closely related to the internal surface of the dura mater and
has exactly the same shape as the Dural sac except where its
arachnoid granulations pierce the dura mater.
 The arachnoid mater is separated from the dura mater by a
capillary space called the subdural space containing a film of fluid.

Page 104 of 284


PROCESSES OF ARACHNOID
A. ARACHNOID VILLI
 These are fine finger-like processes.
 They are covered by specialized mesothelial cells which convey
the CSF to bloodstream, thus leading to the ABSORPTION
OF CSF.

B. ARACHNOID GRANULATIONS [PACCHIONIAN


BODIES]
 With advancing age, the arachnoid villi enlarge in size and
form pedunculated tufts called arachnoid granulations.
 These aggregations of arachnoid villi, clumped together, i.e.,
arachnoid granulations are the large clusters of arachnoid villi.
 Arachnoid granulations like arachnoid villi are concerned with
the ABSORPTION OF CSF.
 They project into the VENOUS LACUNAE OF THE
SUPERIOR SAGITTAL SINUS.

PIA MATER
 Pia mater is a thin transparent vascular membrane which closely
invests the surface of the brain. It is adherent to the surface of the
brain.
 All the blood vessels to the brain run in the subarachnoid space on
the surface of the pia mater before entering the brain.
 The walls of the cavities of the brain (ventricles) are very thin and
made up of only a single layer of its lining epithelium, known as
EPENDYMA
 The pia mater lying on its external surface invaginates into
ventricular cavities as a series of vascular tufts of capillaries which

Page 105 of 284


carry the ependyma before them, thus forming the CHOROID
PLEXUSES of the brain.
 The pial component of choroid plexus is termed as TELA
CHOROIDEA.

SPACES AROUND THE MENINGES


A. EXTRADURAL SPACE
 It is a space lies between the duramater and skull
 Content: MIDDLE MENINGEAL VESSELS
 Its rupture leads to extradural hematoma

B. SUBDURAL SPACE
 It is space between duramater and arachnoid mater
 Content: SUPERIOR CEREBRAL VEINS which drain
into superior sagittal sinus
 Its injury leads to subdural hematoma

Page 106 of 284


C. SUBARACHNOID SPACE
 It is a space between the arachnoid mater and Piamater
 Content: CEREBROSPINAL FLUID
 The arteries and nerves emerging from the brain traverse
this space
SUBARACHNOID SPACE DILATED TO FORM
CISTERNS
 It mainly contains CSF and cerebral vessels.

Page 107 of 284


A. CISTERNA MAGNA
 It is also known as Cerebellomedullary cistern.
 It lies between cerebellum and medulla.
 It receives CSF from fourth ventricle through foramen of
magendie and Luschka.

B. INTERPEDUNCULAR CISTERN
 It lies in interpeduncular fossa.
 Content: CIRCLE OF WILLIS.

C. CISTERNA AMBIENS
 It lies between splenium of corpus callosum and superior
surface of cerebellum.
 Content: GREAT CEREBRAL VEIN OF GALEN

D.CISTERNA SYLVIUS
 It lies in front of each temporal pole.
 Content MIDDLE CEREBRAL ARTERY.

E. CISTERNA PONTIS
 It lies on the ventral surface of pons
 Content: BASILAR ARTERY

APPLIED ANATOMY
1. EXTRADURAL HEMATOMA
 Rupture of extradural space results in extradural hematoma
 DAMAGED STRUCTURE: Middle meningeal artery
(arterial hematoma)

Page 108 of 284


2. SUBDURAL HEMATOMA
 Rupture of subdural space results in subdural hematoma
 DAMAGED STRUCTURE: Veins in the subdural space.

3. CISTERNAL PUNCTURE
 Cerebrospinal fluid can be aspirated through cisterna magna.

Page 109 of 284


DURAL FOLDS
 Duramater has:
1. OUTER ENDOSTEAL LAYER - Attaches to endosteum
2. INNER MENINGEAL LAYER- surrounds the brain

FALX CEREBRI
 Sickle shaped fold of dura mater separates the medial surface of
right and left cerebral hemisphere.

ATTACHMENTS
1. ANTERIOR: Crista galli and Frontal crest of the frontal bone
2. POSTERIOR: Upper surface of tentorium cerebelli
3. SUPERIOR BORDER: Convex margin attached to Cranial vault
4. INFERIOR BORDER: Concave margin is free

DURAL VENOUS SINUS ENCLOSED

Page 110 of 284


1. SUPERIOR SAGITTAL SINUS - along the superior order.
2. INFERIOR SAGITTAL SINUS - along the inferior border.
3. STRAIGHT SINUS - line of attachment of falx cerebri with
tentorium cerebelli.

TENTORIUM CEREBELLI
 It is a TENT-SHAPED FOLD OF THE DURA MATER forming
the roof of the posterior cranial fossa.
 It separates the cerebellum from the occipital lobes of the cerebrum.

MARGINS
1. The INNER FREE MARGIN is U-shaped and encloses the
tentorial notch (incisure) for the passage of the midbrain. The
anterior ends of the concave free margin are attached to the
anterior clinoid processes.

Page 111 of 284


2. The OUTER ATTACHED MARGIN is convex and attached on
each side (from before backward) to the posterior clinoid process,
the posteroinferior angle of the parietal bone, and the lips of
transverse sulci on the occipital bone.

The free and attached margins CROSS EACH OTHER near the apex
of the petrous temporal bone to enclose a triangular area anteriorly
which is pierced by the oculomotor nerve.

SURFACES:
1. The CONVEX UPPER SURFACE slopes to either side from the
median plane. In the median plane, it provides attachment to falx
cerebri
2. The CONCAVE INFERIOR SURFACE provides attachment to
falx cerebelli in its posterior part.

DURAL VENOUS SINUS ENCLOSED: Transverse & superior


petrosal sinus
 Trigeminal or Meckel’s cave is a recess of the dura mater
present in relation to the attached outer margin of the tentorium
cerebelli
 Formation: It is formed by the evagination of the meningeal
layer of the dura mater by two roots of the trigeminal nerve.

FALX CEREBELLI
SHAPE: sickle shaped
Page 112 of 284
EXTENT: From the internal occipital protuberance along the internal
occipital crest to foramen magnum
MARGIN:
1. Anterior concave free margin
2. Posterior convex attached margin

DURAL VENOUS SINUS ENCLOSED: Occipital sinus

DIAPHRAGMA SELLAE
 It is a small circular horizontal fold of the inner layer of the dura
mater forming the roof of the hypophyseal fossa.

Page 113 of 284


ATTACHMENTS
1. ANTERIORLY: tuberculum sellae
2. POSTERIORLY: dorsum sellae and becomes continuous on
each side with the dura mater of the middle cranial fossa.
3. It has a central aperture which provides passage to the stalk of
the pituitary gland.

BLOOD SUPPLY OF THE DURA MATER:


1. In the anterior cranial fossa: by the meningeal branches of the
ophthalmic, anterior, and posterior ethmoidal arteries, and a
branch of the MIDDLE MENINGEAL ARTERY.
2. In the middle cranial fossa: by the middle and accessory
meningeal arteries and by the meningeal branches of the
INTERNAL CAROTID and ASCENDING PHARYNGEAL
ARTERIES.
Page 114 of 284
3. In the posterior cranial fossa: by the meningeal branches of the
VERTEBRAL AND OCCIPITAL ARTERIES.

NERVE SUPPLY OF THE DURA MATER


 Supratentorial dura is supplied by the OPHTHALMIC NERVE.
 Infratentorial dura:
1. In the ANTERIOR CRANIAL FOSSA: by the ANTERIOR and
POSTERIOR ETHMOIDAL NERVES (and receives some twigs
from the maxillary nerve).
2. In the MIDDLE CRANIAL FOSSA: by the MENINGEAL
BRANCH OF THE MAXILLARY NERVE (in the anterior part)
and the MENINGEAL BRANCH OF THE MANDIBULAR
NERVE (nervus spinosus) in the posterior part.
3. In the POSTERIOR CRANIAL FOSSA: by the MENINGEAL
BRANCHES OF THE VAGUS AND HYPOGLOSSAL
NERVES. These are the C1 and C2 fibres carried by the cranial
nerves. The dura mater around the foramen magnum is directly
supplied by the C2 and C3 nerves

Page 115 of 284


FALX CEREBRI
DEFINITION: Sickle shaped fold of dura mater separates the medial
surface of right and left cerebral hemisphere

ATTACHMENTS:
1. ANTERIOR: Crista galli and Frontal crest of the frontal bone
2. POSTERIOR: Upper surface of tentorium cerebelli
3. SUPERIOR BORDER: Convex margin attached to Cranial vault
4. INFERIOR BORDER: Concave margin is free

DURAL VENOUS SINUS ENCLOSED


1. SUPERIOR SAGITTAL SINUS - along the superior order.
2. INFERIOR SAGITTAL SINUS - along the inferior border.
3. STRAIGHT SINUS - line of attachment of falx cerebri with
tentorium cerebelli.

Page 116 of 284


SUPERIOR SAGITTAL SINUS
LOCATION: It is an unpaired Dural venous sinus lies along the
superior attached border of falx cerebri.

ORIGIN: It begins at the crista galli behind the foramen cecum.


COURSE: It then passes backward, and on reaching the internal
occipital protuberance, it deviates usually to the right to become
continuous with the right transverse sinus which in turn becomes
continuous with the right sigmoid sinus. The right sigmoid sinus leaves
the skull through the jugular foramen to continue as the right internal
jugular vein. The size of sinus becomes progressively larger as it passes
backward from the crista galli to the internal occipital protuberance.

TRIBUTARIES:
1. Superior cerebral veins.
2. Parietal emissary vein.
3. Small vein from nasal cavity.
4. Veins of frontal air sinus.

APPLIED ANATOMY
 THROMBOSIS OF SUPERIOR SAGITTAL SINUS: It may be
caused due to spread of infection from dangerous area of nose,
scalp and dipole.

CLINICAL FEATURES
1. Increased intracranial tension
2. Delirium and convulsions

Page 117 of 284


Page 118 of 284
CAVERNOUS SINUS
 Venous space in middle cranial fossa.
 Lies one on the either side of Sella turcica and body of sphenoid
 The 2 sinuses communicate with each other through anterior and
posterior intercavernous sinus.

SURFACES FORMED BY
FLOOR Endosteal duramater
MEDIAL WALL Meningeal duramater
LATERAL WALL Meningeal duramater
ROOF Meningeal duramater

RELATIONS
SUPERIOR
1. Optic chiasma.
2. Optic tract.
3. Internal carotid artery.
4. Anterior perforated substance.

INFERIOR
1. Foramen lacerum.
2. Junction of the body and the greater wing of the sphenoid.

MEDIAL
1. Pituitary gland (hypophysis cerebri).
2. Sphenoid air sinus.

LATERAL
1. Temporal lobe (uncus) of the cerebral hemisphere.
2. Cavum trigeminale containing the trigeminal ganglion.

Page 119 of 284


ANTERIOR
1. Superior orbital fissure.
2. Apex of the orbit.

POSTERIOR
1. Crus cerebri of midbrain.
2. Apex of the petrous temporal bone.

Page 120 of 284


TRIBUTARIES:
1. Superior ophthalmic vein
2. Inferior ophthalmic vein
3. Central vein of retina
4. Superficial middle cerebral vein
5. Inferior cerebral vein
6. Sphenoparietal sinus
7. Frontal trunk of middle meningeal vein

COMMUNICATIONS:
1. Transverse sinus
2. Internal jugular vein through inferior petrous sinus
3. Pterygoid plexus
4. Facial vein
5. Superior sagittal sinus.
6. Internal vertebral venous plexus.

Page 121 of 284


OCULOMOTOR NERVE
FUNCTIONAL COMPONENTS:
GENERAL SOMATIC
Muscles of eyeball for movement
EFFERENTS
GENERAL VISCERAL
Contraction of pupil and accommodation
EFFERENTS
GENERAL SOMATIC Proprioceptive fibres from extraocular muscles to
AFFERENTS mesencephalic nucleus of V nerve.

Page 122 of 284


Page 123 of 284
VISUAL PATHWAY

Page 124 of 284


CHORDA TYMPANI
1. Arises from facial nerve above the stylomastoid foramen
2. Runs upwards, forwards and enters middle ear.
3. It leaves the middle ear through the petrotympanic.
4. Passes medial to spine of sphenoid.
5. Enters infratemporal fossa and joins the lingual nerve and gets
distributed.

FIBRES
PREGANGLIONIC
Submandibular and sublingual salivary
SECRETOMOTOR gland
FIBRES
Anterior two third of tongue except
TASTE FIBRES circumvallate papillae

Page 125 of 284


TRIGEMINAL GANGLION
 The sensory ganglion (gasserian ganglion) of the trigeminal nerve.
 It is homologous with the dorsal root ganglia of spinal nerves.

SHAPE
 Crescentic or semilunar in shape, with its convexity directed
anterolaterally.

SITUATION
 Lies on the trigeminal impression on the anterior surface of petrous
temporal bone near its apex.

MENINGEAL RELATION
 Present in the trigeminal or Meckel’s cave.
 The cave is lined by pia- arachnoid.
 The ganglion along with the motor root is surrounded by CSF.
 the ganglion lies at a depth of about 5cm from the preauricular point.

Page 126 of 284


RELATIONS
MEDIALLY
1. Internal carotid artery.
2. Posterior part of cavernous sinus.
LATERALLY
1. Middle meningeal artery.
SUPERIORLY
1. Parahippocampal gyrus.

INFERIORLY
1. Motor root of trigeminal nerve.
2. Greater petrosal nerve.
3. Apex of the petrous temporal bone.
4. The foramen lacerum.

Page 127 of 284


ROOTS
SENSORY ROOT
 It is the large root made of central processes of ganglion cells.
 Attached to pons at its junction with the middle cerebellar
peduncle.

MOTOR ROOT
 It passes under the ganglion and joins the mandibular nerve at
the foramen ovale.

BRANCHES
 Formed by peripheral processes of ganglion cells.
1. Ophthalmic.
2. Maxillary.
3. Mandibular.
 Emerges from the convexity of the ganglion.

Page 128 of 284


BLOOD SUPPLY: supplied by twigs from:
1. Internal carotid.
2. Middle meningeal.
3. Accessory meningeal arteries.
4. Meningeal branch of the ascending pharyngeal artery.

CLINICAL ANATOMY
TRIGEMINAL NEURALGIA OR CARCINOMATOSIS:
 Intractable facial pain is stopped by injecting alcohol into
trigeminal ganglion.
 Sometimes cutting of sensory root is done.

CILIARY GANGLION
 Peripheral parasympathetic ganglion connected to nasociliary
nerve (ophthalmic division of trigeminal nerve) functionally
connected to oculomotor nerve.

LOCATION:
 Lies near apex of orbit between optic nerve and the tendon of
lateral rectus muscle.

ROOTS:
PARASYMPATHETIC ROOT
 Arises from the nerve to inferior oblique.
 Contains preganglionic fibres

Edinger-Westphal nucleus → preganglionic parasympathetic fibres →


nerve to inferior oblique → relay in ciliary ganglion → postganglionic
fibres → short ciliary nerves → supply ciliaris muscle and sphincter
pupillae.

Page 129 of 284


SENSORY ROOT
 Comes from nasociliary nerve.
 Contains sensory fibres for the eyeball, which do not rely in the
ganglion.

SYMPATHETIC ROOT
 Branch from internal carotid plexus.
 Contains postganglionic fibres arising in the superior cervical
ganglion
 Pass along internal carotid, ophthalmic & long ciliary arteries
 The postganglionic fibres do not relay on ganglion and they pass
further through short ciliary nerves.
 Then they supply blood vessels of eyeball & dilator pupillae.

Page 130 of 284


BRANCHES
 8-10 short ciliary nerves which divide into 15-20 branches
 Branches contain fibres from all three roots of the ganglion.

CLINICAL CORRELATION
CATARACT EXTRACTION: Ciliary ganglion is blocked to produce
dilatation of pupil before cataract extraction.

Page 131 of 284


FORAMEN MAGNUM
 Largest foramen of the skull.
 It is present in the basilar part of occipital bone.
 Opens upwards into the posterior cranial fossa & downwards into
vertebral canal.
 Oval in shape.

STRUCTURES PASSING THROUGH FORAMEN


MAGNUM
THROUGH NARROW ANTERIOR PART
1. Apical ligament of dens
2. vertical band of cruciate ligament
3. Membrana tectoria
THROUGH WIDER POSTERIOR PART

Page 132 of 284


1. Lowest part of medulla oblongata
2. 3 meninges
THROUGH THE SUBARACHNOID SPACE
1. Spinal accessory nerves
2. Vertebral arteries
3. Sympathetic plexus around the vertebral arteries
4. Posterior spinal arteries
5. Anterior spinal artery

ATTACHMENTS
The margins of foramen magnum provide attachment to:
1. Anterior atlanto-occipital membrane, anteriorly
2. Posterior atlanto-occipital membrane, posteriorly
3. Alar ligaments on the roughened medial surface of each occipital
condyle

CLINICAL ANATOMY
 Tonsil of cerebellum project on a side of the medulla oblongata
into the large posterior part of the foramen magnum.

Page 133 of 284


JUGULAR FORAMEN
 It is large and elongated with its long axis directed forwards &
medially.
 It is placed at the posterior end of the Petro- occipital suture.

ANTERIOR WALL – (petrous temporal bone)


 At posterior end of foramen, it is hollowed out to form jugular
fossa. (Fossa is large on right side than on left side.)
 Fossa lodges superior bulb of the internal jugular vein.
 Lateral wall of fossa is pierced by mastoid canaliculus.

MEDIAL END –
1. JUGULAR NOTCH is present –
 Opening in the apex of the notch leads to the cochlear
canaliculus.
2. GLOSSOPHARYNGEAL NOTCH is present –
 Lodges inferior ganglion of the glossopharyngeal nerve.

POSTERIOR BOUNDARY
 Formed by jugular process of occipital bone
STRUCTURES GETTING TRANSMITTED
1. ANTERIOR PART:
 Inferior petrosal sinus.
 Meningeal branch of the ascending pharyngeal artery.
2. MIDDLE PART:
 Glossopharyngeal nerve (9th cranial nerve)
 Vagus nerve (10th cranial nerve).
 Accessory spinal nerve (11th cranial nerve).

11th& 10th cranial nerve is covered by common sheath of dura mater.

3. POSTERIOR PART:
 Internal jugular vein.
 Meningeal branch of the occipital artery.

Page 134 of 284


PTERION
 The pterion is a H shaped suture that can be seen in the lateral
aspect of the skull bone.
 It is present in the anterior part of the temporal fossa
 The four bones that form the H shaped suture are the parietal
bone, the frontal bone, the squamous part of the temporal bone,
and the greater wing of the sphenoid bone

SURFACE MARKING: 4cm above the midpoint of the zygomatic


arch.

SIGNIFICANT RELATION: Deep to it lie the anterior division of


the middle meningeal artery, middle meningeal vein & the stem of
lateral sulcus of brain (sylvian point).

APPLIED ANATOMY
 A sudden blow to the pterion rupture of the anterior division of
the middle meningeal artery  extradural hematoma  may
compress the brain  leads to loss of consciousness or even death

Page 135 of 284


RELATIONS OF CAVERNOUS SINUS
STRUCTURES OUTSIDE THE SINUS:
SUPERIOR:
1. Optic chiasma
2. Optic tract
3. Olfactory tract
4. Internal carotid artery
5. Anterior perforated substance

INFERIOR:
1. Foramen lacerum
2. Junction of the body and the greater wing of sphenoid bone
MEDIAL:
1. Pituitary gland
2. Sphenoidal air sinus
LATERAL:
1. Temporal bone with Uncus

BELOW LATERAL:
1. Mandibular nerve

ANTERIOR:
1. Superior orbital fissure
2. Apex of the orbit

POSTERIOR:
1. Crus cerebri of the midbrain
2. Apex of the petrous part of the temporal bone

Page 136 of 284


STRUCTURES WITHIN THE LATERAL WALL OF SINUS
1. Oculomotor nerve
2. Trochlear nerve
3. Ophthalmic nerve
4. Maxillary nerve
5. Trigeminal ganglion

STRUCTURES PASSING THROUGH THE MEDIAL ASPECT OF


THE SINUS:
1. Internal carotid artery with venous & sympathetic plexus around
it
2. Abducent nerve

CLINICAL ANATOMY:
THROMBOSIS OF CAVERNOUS SINUS
Causes:
1. Severe pain in eye & forehead in area of distribution of ophthalmic
nerve
2. Marked oedema of eyelids, cornea & root of nose with
exophthalmos
PULSATING EXOPHTHALMOS – caused by a communication
between cavernous sinus & internal carotid artery may be due to head
injury.

Page 137 of 284


SIGMOID SINUS
 There are two sigmoid sinuses on either side
 Sigmoid sinus is the continuation of the transverse sinus
 It is s-shaped
 It grooves the inner surface of the mastoid part of the petrous part
of the temporal bone. Here it is separated anteriorly from mastoid
antrum and mastoid air cells by a thin plate of bone.
 It extends from posteroinferior angle of parietal bone to posterior
part of jugular foramen.
 It then continues as the inferior jugular vein

TRIBUTARIES:
1. The mastoid and condylar emissary veins
2. Cerebellar veins
3. Internal auditory vein

APPLIED ANATOMY:
1. MIDDLE EAR INFECTIONS can easily cause thrombosis of the
sigmoid sinus due to its close proximity.
2. During surgical procedures on the middle ear or the mastoid part of
the temporal bone, great care should be taken not to injure the
sigmoid sinus
3. SPREAD OF INFECTION or THROMBOSIS from sigmoid &
transverse sinuses to superior sagittal sinus may cause impaired
CSF drainage into the latter, causing hydrocephalus
4. HYDROCEPHALUS associated with sinus thrombosis following ear
infection is
called as Otitis
hydrocephalus.

Page 138 of 284


FORMATION & DISTRIBUTION OF SPINAL
PART OF ACCESSORY NERVE:
INTRODUCTION:
 The 11th cranial nerve has two roots the cranial and the spinal root.
Of these two the spinal root has more independent course.

FORMATION:
FUNCTIONAL
COMPONENT: Special
visceral efferent
NUCLEI:
 Spinal nucleus of
accessory nerve,
present in the lateral
part of anterior grey
column of the upper
five spinal segments

COURSE:
 The rootlets after
originating from the
spinal cord  unites to form a single trunk in vertebral canal 
ascends to enter the cranial cavity through foramen magnum  it
runs upwards & laterally, crosses jugular tubercle  reaches
jugular foramen where it fuses with the cranial component  the
trunk leaves the cranial cavity through jugular foramen  the
spinal accessory nerve runs between the internal jugular vein and
internal carotid artery  runs in the carotid triangle  pierces
sternocleidomastoid muscle and supplies it  runs in posterior
triangle  finally supplies trapezius muscle.

Page 139 of 284


DISTRIBUTION:
 It supplies,
1. Sternocleidomastoid, the chin turning
2. Trapezius, shrugging muscle

CLINICAL ANATOMY:
 Lesion of the spinal accessory nerve will lead to paralysis of
sternocleidomastoid and the trapezium causing drooping of shoulder
& inability to turn chin to opposite side.

Page 140 of 284


HYPOGLOSSAL NERVE
INTRODUCTION: It is the 12th cranial nerve.
FUNCTIONAL COMPONENT: GSE fibers (arise from the
hypoglossal nucleus), GSA fibres (proprioceptive fibres from tongue
ends in mesencephalic nucleus)

NUCLEI: Hypoglossal nucleus, lies in the floor of 4th ventricle beneath


the hypoglossal triangle.

FORMATION: It arises from the ventral aspect of the medulla, from


the anterolateral sulcus between pyramid & olive.

Page 141 of 284


COURSE AND RELATIONS:
 The nerve comes out of the cranial cavity through the hypoglossal
canal
 It then passes downward to reach the interval between internal
jugular vein and internal carotid artery
 It then passes in front of the internal carotid and external carotid
artery, and the loop of the lingual artery
 It then runs on the superficial surface of the hyoglossus muscle and
then on the superficial surface of the geniohyoid muscle and
supplies the muscles of the tongue

BRANCHES AND DISTRIBUTION:


 Branches containing fibres of hypoglossal nerve proper: It supplies
all intrinsic & extrinsic muscles of the tongue except palatoglossus.
 Branches of hypoglossal nerve that actually contain fibers of C1:
 These fibres join the nerve at the base of the skull.
1. Meningeal branch
2. Descendens hypoglossi
3. Nerve to thyrohyoid
4. Nerve to geniohyoid

Page 142 of 284


APPLIED ANATOMY
• If the nerve is PARALYSED, tongue deviates to the paralysed side.
• INFRANUCLEAR LESION of the hypoglossal nerve produces
paralysis of that side of the tongue with gradual atrophy of paralysed
half of tongue.
• SUPRANUCLEAR LESIONS of hypoglossal nerve causes paralysis
without wasting. On protrusion, the tongue deviates to opposite side.

Page 143 of 284


THIRD VENTRICLE OF BRAIN
 The 3rd ventricle is the cavity of diencephalon.
 It is a midline slit-like cavity situated between two thalami and
part of hypothalamus.
 The cavity of the 3rd ventricle is lined by ciliated columnar
epithelium, the ependyma and traversed horizontally by a mass of
grey matter, the interthalamic adhesion, connecting the two
thalami.

BOUNDARIES
 The 3rd ventricle has anterior wall, posterior wall, roof, floor and two
lateral walls.
1. ANTERIOR WALL is formed from above downward by:
(a) Anterior column of fornix
(b) Anterior commissure
(c) Lamina terminalis.

2. POSTERIOR WALL is formed from above downward by:


(a) Pineal gland
(b) Posterior commissure
(c) Commencement of cerebral aqueduct

3. ROOF is formed by ependyma lining the Tela choroidea of 3rd


ventricle.

4. FLOOR is formed from before backward by:


(a) Optic chiasma
(b) Tuber cinereum and infundibulum
(c) Mammillary bodies

Page 144 of 284


(d) Posterior perforated substance
(e) Tegmentum of the midbrain.

5. LATERAL WALL
(a) Medial surface of thalamus (in its posterosuperior part)
(b) Hypothalamus (in its anteroinferior part)
(c) The hypothalamic sulcus extending from the interventricular
foramen to the cerebral aqueduct.

COMMUNICATIONS
1. ANTEROSUPERIORLY (on each side):
➢ lateral ventricle through interventricular foramen (foramen of
Monroe). This foramen is bounded anteriorly by column of fornix,
posteriorly by tubercle of thalamus.
2. POSTEROINFERIORLY (in the median plane):
➢ 4th ventricle through cerebral aqueduct.

RECESSES
• Recesses are extension of the cavity.
• These are as follows:
1. INFUNDIBULAR RECESS: It is a deep tunnel-shaped recess
extending downward through the tuber cinereum into the
infundibulum.
2. OPTIC (or chiasmatic) RECESS: It is an angular recess
situated at the junction of the anterior wall and the floor of the
ventricle just above the optic chiasma.
3. ANTERIOR RECESS (vulva of the ventricle): It is a
triangular recess which extends anteriorly in front of
interventricular foramen and behind anterior commissure
between the diverging anterior columns of the fornix.

Page 145 of 284


4. SUPRAPINEAL RECESS: It is a fairly capacious blind
diverticulum, which extends posteriorly above the stalk of the
pineal gland and below the Tela choroidea.
5. PINEAL RECESS: It is a small diverticulum which extends
posteriorly between the superior and inferior laminae of the
stalk of the pineal gland.

CLINICAL CORRELATION
 Obstruction of third ventricle: The 3rd ventricle being a narrow slit-
like space is easily obstructed by local brain tumours or congenital
defects. The obstruction results in increased intracranial pressure in
adults and in hydrocephalus in infants.
 The site of obstruction can be found out by CT scans or MRI scans.
 Tumours in lower part of 3rd ventricle produces hypothalamic
symptoms like obesity, diabetes insipidus, disturbance of sleep, etc.

Page 146 of 284


METATHALAMUS
 The metathalamus consists of medial and lateral geniculate
bodies.
 These are small rounded elevations situated on each side of
midbrain, below thalamus.
 The medial and lateral geniculate bodies are relay stations for the
auditory and visual pathways, respectively.

MEDIAL GENICULATE BODY


 Oval elevation on the inferior aspect of the pulvinar of the
thalamus, lateral to the superior colliculus.
 More prominent than the lateral geniculate body.
 Inferior brachium connects the medial geniculate body to inferior
colliculus.
 The connections of medial geniculate body are:
1. AFFERENTS
 Lateral lemniscus
 Fibres from both inferior colliculi
2. EFFERENTS
 It gives rise to acoustic (auditory) radiation going to
auditory area of cortex in the temporal lobe through the
Sublentiform part of internal capsule.

FUNCTION - last rely station on the pathway of auditory


impulses to cerebral cortex.

LATERAL GENICULATE BODY


 Small ovoid prominence visible at the terminal end of the optic
tract.

Page 147 of 284


 Situated anterolateral to the medial geniculate body, below
thalamus.
 Connected to the superior colliculus by the superior brachium.

STRUCTURE
 Six layered structures
 Layers 1,4 & 6 receive contralateral optic fibres
 Layers 2,3 &5 receive ipsilateral optic fibres

AFFERENTS
 Retinal fibres of both the eyes (from temporal half of the retina of the
same side and nasal half of the retina of the opposite side) through
optic tract (lateral root).

EFFERENTS
 Optic radiations going to visual area of cortex in the occipital lobe
through Retrolentiform part of internal capsule.

FUNCTIONS
 Last rely station on the visual pathway to occipital cortex
 The fibres of superior brachium are concerned with the production of
visual reflexes such as turning of head and eyes toward the sudden
flash of light and
constriction of pupil
when light falls on
the retina.

Page 148 of 284


LABELLED DIAGRAM OF PONS AT THE LEVEL
OF FACIAL COLLICULUS:

Page 149 of 284


TRANSVERSE SECTION THROUGH THE UPPER
PART OF PONS:

Page 150 of 284


PONTOCEREBELLAR ANGLE SYNDROME
 Cerebellopontine angle or pontocerebellar angle refers to the space
between cerebellum and pons, filled with CSF
 This area is closely associated with nuclei of few cranial nerves, such
as CN5, CN6, CN7 and CN8 that originate from pons.
 So, any tumor or lesion in this angle will affect these nerves
specifically and exhibit features of nerve palsy of the involved cranial
nerves:
1. Trigeminal (V) nerve palsy - reduced facial sensation
2. Facial (VII) nerve palsy - facial droop / weakness
3. Vestibulocochlear (VIII) nerve palsy - unilateral
sensorineural hearing loss, nystagmus, ataxia

MICROSCOPIC STRUCTURE OF CEREBELLUM


 The cerebellar cortex consists of three layers: outer molecular,
middle purkinje cell layer and innermost granular layer

MOLECULAR LAYER
NERVE FIBERS-
 Dendrites of the Purkinje cells
 Axons of granule cells
 Afferent fibers from inferior olivary nucleus that synapses with
dendrites of the Purkinje cells

NERVE CELLS-
 Superficially located Stellate cells
 Deeply located Basket cells

Page 151 of 284


PURKINJE CELL LAYER
 One layer of large flask-shaped Purkinje cells
 Axons of Purkinje cells are the only output of the cerebellum

GRANULE CELL LAYER


It consists of
1. GRANULE CELLS: Small, numerous and the only excitatory
neurons.
2. GOLGI CELLS: Large and less numerous than granule cells.
They are inhibitory neurons.

SCHEMATIC DIAGRAM OF THE LAYERS AND THE


CELLS

Page 152 of 284


HISTOLOGICAL DIAGRAM OF THE LAYERS

Page 153 of 284


RHOMBOID FOSSA
• The floor of the fourth ventricle is called the rhomboid fossa due to
its shape (like a rhomboid – diamond)
• It is formed by the posterior surface of pons and medulla.
• This diamond shaped fossa is divided into an upper triangle and a
lower triangle.

BOUNDARIES:
1. UPPER TRIANGLE- superior cerebellar peduncles,
2. LOWER TRIANGLE- gracile and cuneate tubercle along with
the inferior cerebellar peduncle

Page 154 of 284


FEATURES:

MEDIAN SULCUS
 Runs longitudinally and separates the floor into symmetrical
halves. It extends from the aperture of the midbrain above to the
beginning of the central canal below.

STRIA MEDULLARIS
 These fibers cross the floor at its widest part. They emerge from
the median sulcus  run on the floor laterally  enters the inferior
cerebellar peduncle.

MEDIAL EMINENCE:
 Longitudinal elevations on either side of median sulcus.

SULCUS LIMITANS:
 Limits medial eminence laterally.

VESTIBULAR AREA:
 Region lateral to the sulcus limitans. Vestibular nucleus is
present below it, hence the name.

SUPERIOR FOVEA:
 Widened triangular depression at upper end of sulcus limitans.

LOCUS CERULEUS:
 Flattened-out part of sulcus limitans, above the superior fovea.
Bluish grey, due to substantia ferruginea, melanin pigment
containing group of neurons.

INFERIOR FOVEA:
 Small depression at the lowermost part of the sulcus limitans.

Page 155 of 284


FACIAL COLLICULUS:
 Oval elevation on medial eminence at the level of superior fovea.
It is the swelling produced by the turning of the motor fibres of
facial nerve around nucleus of abducent nerve.

HYPOGLOSSAL TRIANGLE:

 Below the level of inferior fovea, the sulcus limitans will run
towards the midline. Due to this the medial eminence is divided
into hypoglossal triangle (above) and vagal triangle (below). It
overlies the hypoglossal nuclei and nucleus intercalatus.

VAGAL TRIANGLE:

 It overlies nuclei of vagus, glossopharyngeal and cranial accessory


nerve.

FUNICULUS SEPARANS:

 Narrow translucent ridge in vagal triangle.

AREA POSTREMA:

 Area between funiculus separans and gracile tubercle.

TAENIA:

 Narrow white ridge at the inferolateral margins of fossa.

OBEX:

 Present at inferior angle of ventricle and is formed by fusion of the


two taenia.

Page 156 of 284


Page 157 of 284
CEREBELLAR PEDUNCLE
 They are bundles of fibers carrying afferents to and efferents from
the cerebellum
 There are three cerebellar peduncles, superior, middle and
inferior. The middle one is the largest of all.

CONNECTIONS
1. SUPERIOR CEREBELLAR PEDUNCLE: cerebellum and
midbrain
2. MIDDLE CEREBELLAR PEDUNCLE: cerebellum and
pons
3. INFERIOR CEREBELLAR PEDUNCLE: cerebellum and
medulla

CONTENTS
1. SUPERIOR CEREBELLAR PEDUNCLE:
 Mainly efferents- from dentate nucleus of cerebellum to red
nucleus, thalamus and cerebral cortex of opposite side. It is
the main efferent pathway from the cerebellum.
2. MIDDLE CEREBELLAR PEDUNCLE:
 Mainly afferents- from the pontine nucleus of the opposite
side.
3. INFERIOR CEREBELLAR PEDUNCLE:
 Mainly afferents- from the spinal cord, the olivary nuclei,
reticular formation, vestibular nuclei.
 Few efferents- to vestibular nuclei and reticular formation.

Page 158 of 284


INTERPEDUNCULAR FOSSA
 Rhomboidal space

BOUNDARIES
1. Anterior-optic chiasma and optic tracts
2. Posterior-pons
3. Sides-crus cerebri of cerebral peduncles

CONTENTS
1. Mammillary bodies (2 spherical bodies)
2. Tuber cinereum (a raised area of grey matter lying anterior to
mammillary bodies)
3. Infundibulum (a narrow stalk connecting hypophysis cerebri with
the tuber cinereum)
4. Posterior perforated substance (layer of grey matter in the angle
between the crus cerebri)
5. Occulomotor nerve (emerges immediately dorsomedial to the
corresponding crus)

Page 159 of 284


DENTATE NUCLEUS
 The dentate nucleus is the most prominent of
the intracerebellar nuclei
 Largest in primates, especially in humans.
 It is the nucleus of neocerebellum and
therefore receives afferent fibres from it.

NEOCEREBELLUM DENTATE NUCLEUS RED NUCLEUS AND THALAMUS

 The superior cerebellar peduncle mainly


consists of efferent fibres passing from the dentate nucleus to the
red nucleus, thalamus and cerebral cortex of the opposite side. It
is the principal efferent pathway from the cerebellum and its
fibres arise mainly
in the dentate
nucleus.

Page 160 of 284


ENUMERATE THE IMPORTANT SIGNS AND
SYMPTOMS SEEN IN CEREBELLAR LESION
CEREBELLAR SYNDROME:
 Cerebellar lesions due to trauma, vascular
lesions, tumours etc produce a number of signs and symptoms,
which together constitute the cerebellar syndrome.

CHARACTERISTICS SIGNS AND SYNDROMES:


1. Swaying toward the side of lesion on walking
2. Falling on the side of the lesion on closing the
eyes (ROMBERG’S SIGN)
3. Intention tremors at the end of a voluntary act
and disappear with rest
4. Generalised muscular hypotonia leading to
a. Staggering gait (the patient walks with legs
apart)
b. Ataxia (inability to maintain balance while
walking)
5. Adiadochokinesis (i.e., inability to perform
rapidly alternating opposite movements)
6. Dysarthria or scanning speech (i.e., the speech
is slurred, monotonous with pauses at wrong places)
7. Nystagmus (i.e., involuntary “to and fro”
oscillatory movements of the eyeballs while looking to either side
due to defective postural fixation of the conjugate gaze)

Page 161 of 284


LATERAL MEDULARY SYNDROME:
 Is also called lateral medullary syndrome of
Wallenberg
 Due to thrombosis of posterior inferior
cerebellar artery, thus affecting a wedge-shaped area on the
dorsolateral aspect of the medulla and the inferior surface of the
cerebellum

SIGNS AND SYMPTOMS


1. Giddiness, due to involvement of vestibular
nuclei
2. Ipsilateral ataxia, due to involvement of
inferior cerebellar peduncle and cerebellum
3. Ipsilateral paralysis of muscles of palate,
pharynx and larynx due to involvement of nucleus ambiguous
4. Ipsilateral loss of pain and temperature
sensation over the face, due to involvement of spinal nucleus
and tract of trigeminal nerve
5. Contralateral loss of pain and temperature
sensation in the trunk and limbs due to involvement of
spinothalamic tract

Page 162 of 284


MEDIAL MEDULLARY SYNDROME
 It is also called Dejerine anterior bulbar syndrome.

CAUSE: blockage of anterior spinal artery.


DAMAGE TO: paramedian region of medulla.
FEATURES:
1. Contralateral hemiplegia/ paralysis of arm & leg – damage to
pyramid of medulla.
2. Ipsilateral paralysis of muscles of tongue – injury to
hypoglossal nerve (XII cranial nerve).

Page 163 of 284


3. Contralateral loss of sense of vibration & position – damage to
medial lemniscus.

T.S. OF MID-BRAIN AT THE LEVEL OF


INFERIOR COLLICULI
GREY MATTER STRUCTURES
CENTRAL (periaqueductal) GREY MATTER contains:
I. NUCLEUS OF TROCHLEAR NERVE
a. Located: ventro medial part.
II. MESENCEPHALIC NUCLEUS OF TRIGEMINAL NERVE

Page 164 of 284


a. Located: lateral part.
b. Made up of unipolar cells.
c. Receives proprioceptive impulses from –muscles of
mastication, facial & ocular muscles, teeth and
temporomandibular joint.

INFERIOR COLLICULUS:
1. Afferents: lateral lemniscus.
2. Efferents: medial geniculate body.
3. Helps in localising the source of sounds
4. In past considered as the centre for auditory reflexes.

SUBSTANTIA NIGRA:

Page 165 of 284


1. Deeply pigmented nerve cells.
2. Concerned with muscle tone.

WHITE MATTER STRUCTURES


I. CRUS CEREBRI CONTAINS:
a. Middle ⅔rd - corticospinal tract.
b. Medial ⅙the - Frontopontine fibres.
c. Lateral ⅙the -temporopontine, parietopontine &
occipitopontine fibres.
II. TEGMENTUM CONTAINS: Ascending tracts.
a. The lemnisci are arranged in a band like a necklace.
 In the order from medial to lateral side as medial,
trigeminal, spinal & lateral lemniscus.
b. Decussation of superior cerebellar peduncles:
 In the median plane.
c. Medial longitudinal bundle:
 Close to the trochlear nucleus.
d. Tectospinal tract.
e. Rubrospinal tract.
III. TROCHLEAR NERVE:
a. Passes laterally and dorsally around the central grey
matter.
b. Decussates in superior medullary velum.
c. Emerges lateral to the frenulum veli.

T. S. OF MIDBRAIN AT THE LEVEL OF


SUPERIOR COLLICULI
GREY MATTER
I. CENTRAL GREY CONTAINS:
a. Oculomotor nerve nuclei & Edinger – Westphal
nucleus.
 In ventromedial part.
Page 166 of 284
 Oculomotor nuclei are very close to each other.
b. Mesencephalic nucleus of trigeminal nerve:
 Lateral part.
II. SUPERIOR COLLICULUS:
a. Afferents: retina (visual) & Various other centres.
b. Efferents: tectospinal tract.
c. Controls reflex movements of the eyes & head and neck in
response to visual stimuli

III. PRETECTAL NUCLEUS:


a. Deep to superolateral part of the superior colliculus.
b. Afferents: lateral roots of optic tract.
c. Efferents: to Edinger- Westphal nuclei of both sides.
d. Part of the pathway for reflex and consensual reflex.

Page 167 of 284


IV. RED NUCLEUS:
a. Afferents: superior cerebellar peduncle, globus pallidus,
subthalamic nucleus & cerebral cortex.
b. Efferents: rubrospinal tract, reticular formation, thalamus,
olivary nucleus, subthalamic nucleus etc.
c. Inhibitory influence on muscle tone.
V. SUBSTANTIA NIGRA:
a. Deeply pigmented nerve cells.
b. Concerned with muscle tone.

WHITE MATTER
I. CRUS CEREBRI CONTAINS:
a. Middle ⅔rd - Corticospinal tract.
b. Medial the - Frontopontine fibres.
c. Lateral ⅙the - Temporopontine, parietopontine &
occipitopontine fibres.
II. TEGMENTUM CONTAINS:
a. The lemnisci are arranged in a band like a necklace.
➢ In the order from medial to lateral side as medial,
trigeminal & spinal lemniscus.
b. Decussation of rubrospinal tracts – ventral tegmental
decussation.
c. Medial longitudinal bundle.
d. Emerging fibres of oculomotor nerve.
III. Tectum shows the posterior commissure connecting the two
superior colliculi.

CLINICAL ANATOMY
i. PARINAUD'S SYNDROME:
 Lesion of superior colliculi.
 Features:

Page 168 of 284


1. Weakness of upward gaze.
2. Vertical nystagmus.
ii. ARGYLL ROBERTSON PUPIL:
 Lesion in vicinity of pretectal nucleus.
 Light reflex – lost.
 Accommodation reflex – retained.
iii. WEBER’S SYNDROME:
 Lesion involves 3rd nerve nucleus & corticospinal tract.
 Contralateral hemiplegia (corticospinal tract)
 Pupil points downwards & laterally (paralysis of 3rd nerve).
iv. BENEDIKT'S SYNDROME:
 Loss of proprioception (medial lemniscus).
 Pupil points downwards & laterally (paralysis of 3rd nerve).
 Contralateral twitching and tremors (red nucleus & superior
cerebellar peduncle).

Page 169 of 284


MEDIAL LONGITUDINAL BUNDLE
 MLF is a heavily myelinated composite tract.
 It is found in Paramedian plane of brainstem.
 The fasciculus retains its position relative to central grey matter
throughout the midbrain, pons and upper medulla.
 But it is displaced ventrally by pyramidal decussation having
corticospinal fibres.

EXTENSION:
1. CRANIALLY - Interstitial cells of Canal (at junction of midbrain
and diencephalon)
2. CAUDALLY - continues with anterior intersegmental fasciculus
of spinal cord.

NUCLEI (FIBRES ARISE FROM)


 Mainly Vestibular nuclei (all four), also lateral lemniscus,
interstitial nucleus of Canal
 It interconnects nuclei of 3rd, 4th, 6th, 7th, Spinal accessory,
Edinger Westphal preganglionic, reticular.

FUNCTIONS:
 Coordinating conjugate eye movements and associated movements of
head and neck in response to stimulation of the 8th nerve.

Page 170 of 284


Page 171 of 284
MLF SYNDROME (INTERNUCLEAR
OPHTHALMOPLEGIA)
 Occurs due to lesion of upper part of pons in the region between
abducent and oculomotor nuclei.
 MLF Syndrome mostly seen with multiple sclerosis with following
Clinical features:
1. Isolated paralysis of medial rectus muscle. (On same side of
lesion).
2. Monocular horizontal nystagmus (on opposite side of Lesion).

Page 172 of 284


HORNS OF LATERAL VENTRICLE

DEFINITION
 Lateral ventricle is two ‘C shaped’ cavity present on each side of
cerebral hemisphere.
 It is lined by ependyma and filled with CSF.

HORNS: They are extensions of lateral ventricle


DIRECTION OF
NAME LOBE
EXTENSION
ANTERIOR
Forward Frontal
HORN
CENTRAL BODY - Parietal
POSTERIOR
Backward Occipital
HORN
INFERIOR
Downward Temporal
HORN

Page 173 of 284


BOUNDARIES
ANTERIOR HORN
1. ROOF: Body corpus callosum
2. FLOOR: Rostrum of corpus callosum
3. MEDIAL WALL: Septum pellucidum
4. LATERAL WALL: Head of caudate nucleus
5. ANTERIOR WALL: Genu of corpus callosum

POSTERIOR HORN
1. ROOF, LATERAL WALL, FLOOR:
a. Tapetum
b. Optic radiation
c. Inferior longitudinal fasciculus
2. MEDIAL WALL:
a. Bulb of posterior horn (forceps major)
b. Calcar Avis

Page 174 of 284


INFERIOR HORN
Largest and longest horn
1. ROOF:
a. Tail of caudate nucleus
b. Tapetum
2. FLOOR:
a. Collateral eminence
b. Hippocampus

3. BODY:
a. Roof: Body of corpus callosum
b. Floor:
1. Body of caudate nucleus
2. Stria terminalis
3. Thalamo striate vein
4. Upper surface of thalamus
5. Choroid plexus
6. Fornix

Page 175 of 284


FOURTH VENTRICLE
 Tent like cavity in hind brain

LOCATION
 Posterior cranial fossa
 Lined by ependyma filled with CSF

FEATURES
1. Recess
2. Angles
3. Boundaries

RECESS

ANGLES
 It has four angles.
 Superior, inferior, 2 laterals
1. Superior angle continues with cerebral aqueduct of mid
brain
2. Inferior angle continues with central of medulla
3. Lateral angle continuous with lateral recess

Page 176 of 284


BOUNDARIES
1. SUPEROLATERALLY: superior cerebellar peduncle
2. INFEROLATERALLY: inferior cerebellar peduncle
3. ROOF:
 Tent shaped
 UPPER PART of the roof: superior medullary velum
between the two superior cerebellar peduncles
 LOWER PART of the roof: Inferior medullary velum
between the two inferior cerebellar peduncles
 TELA CHOROIDEA: Double layered fold of pia matter
 CHOROIDAL PLEXUS: capillary plexus in the Tela
choroidea

4. FLOOR:
 MEDIAN SULCUS divides the floor into right and left halves
 On either side of median sulcus there is a longitudinal
elevation called MEDIAN EMINENCE

Page 177 of 284


 Median eminence is bound laterally by SULCUS LIMITANS
 The region lateral to sulcus limitans overlies the vestibular
area, hence it is termed as VESTIBULAR AREA
 The upper end of sulcus limitans: SUPERIOR FOVEA
 Above superior fovea sulcus limitans flattens out and presents
bluish area called LOCUS CERULEUS (melanin and
noradrenaline)
 The lower end of
sulcus limitans
presents a small
depression called
INFERIOR FOVEA
 On either side of
median eminence
shows an oval
swelling FACIAL
COLLICULI
(winding of facial
nerve around
abducent nerve)
 The inferior fovea of
sulcus limitans
descends obliquely
towards the median
sulcus. And divides the medullary part of the floor into
HYPOGLOSSAL and VAGAL TRIANGLE.
 Vagal triangle is crossed by a narrow ridge called
FUNICULUS SEPARENS. Area below funiculus separans is
called AREA POSTREMA.
 Inferolateral margin of 4th ventricle is marked by narrow ridge:
TAENIA
 The taenia meet to form a fold called OBEX.

Page 178 of 284


MEDULLA: PYRAMIDAL DECUSSATION

 POSTERIOR ASPECT: narrow strip of nucleus Gracilis and


nucleus cuneatus
 APEX of the posterior
horn gets swollen to
form the spinal
nucleus and spinal
tract of trigeminal
nerve
 DECUSSATION
OF
CORTICOSPINAL
FIBRES: 90 % of
fibres runs backwards
and laterally and
continues as lateral
corticospinal tract
while the other (10%) travel as anterior corticospinal tract.
 MEDULLARY PYRAMIDS are ridge like structures present on
the ventral aspect of medulla.
 As a result of decussation anterior horn detach and forms spinal
nucleus of accessory nerve and supraspinal nucleus of 1st
cervical nerve
 NETWORK OF FIBRES: Reticulate formation

Page 179 of 284


APPLIED

WALLENBERG (LATERAL MEDULLARY SYNDROME)

CAUSE: Occlusion of vertebral artery > Posterior inferior cerebellar


artery

IPSILATERAL SIGNS AND SYMPTOMS:


1. Loss of pain and temperature over half face (trigeminal nerve)
2. Cerebellar ataxia of limbs (spinocerebellar tract)
3. Nystagmus vertigo nausea (vestibular nucleus)
4. Horner’s syndrome: ptosis, anhidrosis, miosis (sympathetic tract)
5. Loss of taste (Nu. Tactus solitarius)

CONTRALATERAL SIGNS AND SYMPTOMS:


Loss of pain and temperature over half of the body (lateral
spinothalamic tract)

MEDIAL MEDULLARY SYNDROME

CAUSE: occlusion of vertebral / anterior spinal artery or basilar artery


IPSILATERAL:
 Flaccid (LMN) paralysis and atrophy of one half of the tongue
(hypoglossal)
CONTRALATERAL:
 Spastic paralysis (UMN) paralysis and loss of proprioception,
tactile discrimination, vibration sense

Page 180 of 284


MEDULLA (OLIVARY LEVEL):

CENTRAL GREY MATTER is spread over the floor of fourth


ventricle and contains several CRANIAL NERVE NUCLEI.
1. Hypoglossal
2. Dorsal nucleus of vagus
3. Nucleus Tractus Solitarius

On EITHER SIDE OF MIDLINE:


1. Medial longitudinal bundle
2. Tectospinal
3. Rubrospinal

NUCLEUS AMBIGUOUS gives origin to cranial nerve 9, 10, 11.

Page 181 of 284


ARCUATE NUCLEUS receives fibres from cortex and send it to
cerebellum

OLIVARYNUCLEUS:
1. The INFERIOR OLIVARY NUCLEUS (or 'complex') which is a
part of the olivo-cerebellar system and is mainly involved in
cerebellar motor-learning and function.
2. The SUPERIOR OLIVARY NUCLEUS, considered part of the pons
and part of the auditory system, aiding the perception of sound.

Page 182 of 284


TRANSVERSE SECTION THROUGH MIDDLE OF
MEDULLA PASSING THROUGH SENSORY
DECUSSATION

GREY MATTER
 Lateral to cuneate nucleus there is accessory cuneate nucleus
which relays unconscious proprioceptive fibres from the upper
limbs. It is equivalent to nucleus dorsalis/Clarke’s column.
 The nucleus of the spinal tract of trigeminal nerve is also separate
from the central grey matter.
 The lower part of the inferior olivary nucleus is seen.
 The central grey matter contains the following-
1. HYPOGLOSSAL NUCLEUS - an elongated nucleus about
2 cm long, supplies muscles of tongue except palatoglossus.
2. DORSAL NUCLEUS OF VAGUS - gives preganglionic and
parasympathetic fibres to heart, smooth muscles and glands
of respiratory and alimentary system.
3. NUCLEUS OF TRACTUS SOLITARIES - receives taste
fibres.

Page 183 of 284


WHITE MATTER
 The nucleus Gracilis and cuneatus give rise to internal arcuate
fibres. These fibres cross to the opposite side where they form a
paramedian band of fibres called medial lemniscus. In the lemniscus,
the body is represented with the head posteriorly and the feet
anteriorly.
 The pyramidal tracts lie anteriorly.
 The medial longitudinal bundle lies posterior to medial lemniscus.
 The spinocerebellar, lateral spinothalamic and other tracts lie in
anterolateral area.
 Emerging fibres of XII nerve(hypoglossal).

Page 184 of 284


DIENCEPHALON- DEFINITION AND
SUBDIVISIONS
 The diencephalon is a part of brain situated cranial to the
midbrain and is more or less completely surrounded by the
cerebrum.
 The only part of diencephalon which is exposed to the surface is at
the base of the brain in the region of interpeduncular fossa.
 The cavity within the diencephalon is termed the 3RD
VENTRICLE.
 It communicates on either side with
the lateral ventricle of the cerebral
hemisphere.
 The cavity of the 3rd ventricle
divides the diencephalon into two
(right and left) symmetrical halves.

DIVISIONS AND SUBDIVISIONS

 The diencephalon is divided into two major parts: pars dorsalis and
pars ventralis.
 These subdivisions are seen in midsagittal view of the brain and are
separated from each other by a shallow groove, the hypothalamic
sulcus, which extends from interventricular foramen to the rostral
end of the cerebral aqueduct of the midbrain.
1. PARS DORSALIS
lies above (dorsal) the
hypothalamic sulcus
and consists of: (a)
thalamus, (b)
metathalamus, and (c)
epithalamus.

Page 185 of 284


2. PARS VENTRALIS lies below (ventral) the hypothalamic
sulcus and consists of: (a) subthalamus and (b) hypothalamus.
 Thus, the diencephalon is divided into five parts: thalamus,
metathalamus, epithalamus, subthalamus, and hypothalamus.
 Each of these parts has further subdivisions.

Fig: diencephalon and its parts.

Page 186 of 284


4THALAMUS - DEFINITION AND NUCLEI
 Anatomically, thalamus is a large ovoid mass of grey matter lying
above the midbrain, from which it is separated by a small amount
of neural tissue, the SUBTHALAMUS.
 There are two thalami situated one on each side of a slit-like
cavity, the 3RD VENTRICLE.

Fig. Thalamus and hypothalamus in sagittal view.

NUCLEI OF THALAMUS
1. NUCLEI IN THE ANTERIOR PART
 The nuclei in this part are collectively referred to as
ANTERIOR NUCLEUS.
2. NUCLEI IN THE MEDIAL PART
 Nuclei in medial part consist of a large MEDIAL DORSAL
NUCLEUS and a small MEDIAL VENTRAL NUCLEUS.
3. NUCLEI IN THE LATERAL PART
 The lateral part is divided into dorsal and ventral parts.
 The dorsal part is subdivided craniocaudally into three
nuclei: (a) lateral dorsal (LD), (b) lateral posterior (LP), and
(c) a large caudal nuclear mass, the pulvinar (P). These
nuclei are termed DORSAL TIER OF NUCLEI.

Page 187 of 284


 The ventral part is also subdivided craniocaudally into three
nuclei: (a) ventral anterior (VA), (b) ventral lateral (VL) or
ventral intermediate (VI), and (c) ventral posterior (VP).
 These nuclei are termed VENTRAL TIER OF NUCLEI.
The ventral posterior nucleus (VP) is further subdivided into
a lateral part, the ventral posterolateral nucleus (VPL) and a
medial part, the ventral posteromedial nucleus (VPM).

FIG: nuclei of thalamus

Page 188 of 284


PINEAL GLAND
 Pineal gland is a midline cone-shaped reddish grey structure (only
3 mm - 5 mm in size) occupying the vertical groove between the
two superior colliculi below the splenium of corpus callosum.
 It projects back from the posterior wall of the 3rd ventricle, below
the splenium of the corpus callosum.
 It has a stalk which divides into two laminae.
 The ventral (or inferior) lamina continues with the posterior
commissure and the dorsal (or superior) lamina continues with the
habenular commissure.
 The extension of the cavity of the 3rd ventricle between the two
laminae is termed pineal recess.

STRUCTURE AND FUNCTIONS:


 The pineal gland is a neuroendocrine gland and consists of
parenchymal cells, called PINEALOCYTES and NEUROGLIAL
CELLS.

Page 189 of 284


 The pinealocytes secrete a hormone called MELATONIN.
 The calcium phosphates and carbonates are deposited in the gland
with age in the form of multilaminar corpuscles called
CORPORA ARENACEA or BRAIN SAND.
 They are often seen as tiny shadows in radiographs of the skull. A
displaced calcified pineal gland indicates a space-occupying lesion
within the brain.
 Pineal secretions including melatonin have an inhibitory effect on
other endocrine glands and gonads.

UNIQUE FEATURES
 Pineal gland is the only part of the brain which has no neural
tissue in it.
 It is the only part of the brain which is supplied by a nerve
(nervus conarii) which arises from outside the brain from superior
cervical sympathetic ganglion in the neck.

Page 190 of 284


PARTS, DEEP NUCLEI & ARTERIAL SUPPLY OF
CEREBELLUM
PARTS:
 Cerebellum consist of two cerebellar hemispheres that are united
to each other by median vermis.
 The cerebellum is subdivided into numerous small parts by fissures.
 Each fissure cuts both vermis and hemispheres.
1. The horizontal fissure separates the superior surface and
inferior surface.
2. The primary fissure separates the anterior lobe from the
middle lobe on the superior surface of cerebellum.
3. The posterolateral fissure separates middle lobe from
flocculonodular lobe on the inferior surface.

PARTS OF VERMIS WITH RESPECTIVE PART OF


CEREBELLAR HEMISPHERE
SUBDIVISIONS OF THE CEREBELLAR
PARTS OF VERMIS
HEMISPHERE

Lingula -
Central lobule Ala
Culmen Quadrangular lobe
Declive Simple lobule
Folium Superior semilunar lobule
Tuber Inferior semilunar lobule
Pyramid Biventral lobule
Uvula Tonsil
Nodule Flocculus

Page 191 of 284


DEEP NUCLEI
 These nuclei lie close to the roof of IVth ventricle. So, they are also
called as “roof nuclei.”
 Mass of grey matter embedded in central core of white matter
constitute cerebellar nuclei: LATERAL
1. Dentate nucleus
2. Nucleus interpositus
3. Fastigial nucleus
MEDIAL

DENTATE NUCLEUS
 Lies in center of each cerebellar hemisphere
 Resembles a crumpled purse with hilum directed medially.
 Receives fibres from lateral region.

NUCLEUS INTERPOSITUS
 Emboliform nuclei + Globose nuclei = Nucleus interpositus
 Lie medial to dentate nucleus.
 Receives fibres from paravermal regions.

Page 192 of 284


FASTIGIAL NUCLEUS
 Lie close to the midline in anterior part of superior vermis.
 Receives fibres from vermis.

ARTERIAL SUPPLY FOR CEREBELLUM


 Supplied by three pairs of cerebellar arteries:
1. SUPERIOR CEREBELLAR ARTERY
 Branch of basilar artery
 Supply superior surface of cerebellum.

2. ANTERIOR INFERIOR CEREBELLAR ARTERY


 Branch of basilar artery
Page 193 of 284
 Supplies anterior part of inferior surface of cerebellum.

3. POSTERIOR INFERIOR CEREBELLAR ARTERY


 Branch of vertebral artery.
 Supply posterior part of inferior surface of cerebellum.

Page 194 of 284


CALCARINE SULCUS
 The calcarine sulcus (or calcarine fissure) is an anatomical
landmark located at the caudal end of the medial surface of
the brain of humans and other primates.
 Its name comes from the Latin "calcar" meaning "spur".
 It is very deep and known as a complete sulcus.
 The calcarine sulcus begins near the occipital pole in two
converging rami and runs forward to a point a little below
the splenium of the corpus callosum, where it is joined at an acute
angle by the medial part of the parieto-occipital sulcus.
 The anterior part of this sulcus gives rise to the prominence of
the calcar Avis in the posterior cornu of the lateral ventricle.
 The calcarine sulcus is where the primary visual cortex (V1) is
concentrated.
 The central visual field is located in the posterior portion of the
calcarine sulcus and the peripheral visual field in the anterior
portion.

Page 195 of 284


CONNECTIONS OF BASAL GANGLIA
CONNECTIONS OF CORPUS STIATUM:
AFFERENT CONNECTIONS:
 Entire cerebral cortex via corticostriate fibres.
 Intralaminar nuclei of thalamus via thalamostriate fibres.
 Pars compacta of substantia nigra via nigrostriatal fibres
 Noradrenergic fibres received from locus ceruleus.
 Serotoninergic fibres received from raphe nucleus.

EFFERENT CONNECTIONS:
1. FASCICULUS LENTICULARIS- arise from internal segment of
globus pallidus and enters the subthalamic region.
2. ANSA LENTICULARIS- arise from both internal and outer
segment of globus pallidus and enters subthalamic region where it
meets the dentarubrothalamic fibres and the fasciculus
lenticularis. The union of the three tracts is called thalamic
fasciculus, which terminates in ventral anterior, ventral lateral
and subthalamic nuclei of thalamus.
3. SUBTHALAMIC FASCICULUS consists of reciprocal
connections between the globus pallidus and nucleus
subthalamus.
4. Some fibres from globus pallidus also pass to substantia nigra
(pallidonigral fibres).

CONNECTONS OF AMYGDALOID BODY:


AFFERENTS: From olfactory tract

Page 196 of 284


EFFERENTS: It gives rise to stria terminalis which ends in anterior
commissure, the anterior perforated substance and in hypothalamic
nuclei.

MAJOR CONNECTIONS OF BASAL NUCLEI


CEREBRAL CORTEX
PREMOTOR AND MOTOR CORTEX

Thalamus
Striatum Intralaminar VA and VL

Substantia
nigra Pallidum Subthalamus

Red nucleus Reticular formation

Spinal cord

Connections of corpus striatum

Relations of corpus striatum with internal capsule

Relations of caudate nucleus with cavity of lateral ventricle and thalamus

Page 197 of 284


Page 198 of 284
LABELLED DIAGRAM OF SUPEROLATERAL
SUFACE OF CEREBRUM, INDICATING MAJOR
FUNCTIONAL AREAS

Page 199 of 284


STRUCTURES FORMING THE LIMBIC SYSTEM
 The structures forming the limbic system are interposed between
the superolateral surfaces of the diencephalon and the
inferomedial surfaces of the two cerebral hemispheres.

 LIMBIC LOBE, consisting of cingulate gyrus, isthmus,


Parahippocampal gyrus and uncus (anterior part of the
Parahippocampal gyrus).
 HIPPOCAMPAL FORMATION which includes
hippocampus (cornu ammonis), dentate gyrus, gyrus fasciolaris
and indusium griseum.

Page 200 of 284


SUBCORTICAL NUCLEI
1. Amygdaloid nuclear complex (also called amygdaloid body).
2. Septal region and nuclei.
3. Olfactory areas
4. Hypothalamus especially the mammillary bodies.
5. Anterior nucleus of thalamus.

Page 201 of 284


COMPONENTS OF BASAL GANGLIA
Anatomically, the term basal ganglia include:
a. Corpus striatum,
b. Claustrum, and
c. Amygdaloid body.

CORPUS STRIATUM:

CLAUSTRUM:
 Claustrum is a thin saucer-shaped mass of grey matter situated
between the putamen and insula.
 It is considered as a detached part of the insula. Its connections
and functions are not known.
Page 202 of 284
AMYGDALOID BODY:
 Amygdaloid body is an almond-shaped mass of grey matter in the
temporal lobe, lying anterosuperior to the tip of inferior horn of
lateral ventricle.
 It is situated deep to uncus which serves as a surface which serves
as a surface landmark for its location.
 Developmentally it is related to basal nuclei but functionally it is
included in the limbic system and therefore shares its functions.

Page 203 of 284


PARKINSONISM
 This disease usually occurs after 50 years of age due to deficiency of
the neurotransmitter dopamine in the corpus striatum following a
lesion in substantia nigra and its projections (i.e., nigrostriatal
fibres).
 It is characterized by the triad of symptoms, viz. tremor, akinesia
and rigidity.

CHARACTERISTIC FEATURES OF PARKINSONISM


 Resting tremors, i.e., a slight shaking of hands when person is not
performing a task. The tremors are diminished with movement and
exaggerated by emotional excitation.
 Lead-pipe or cogwheel type of muscular rigidity due to increased
muscle tone (in contrast to clasp-knife rigidity of upper motor neuron
lesions).
 Pill-rolling movements of hands, consisting of circular movements of
the opposed thumb.

FUNCTIONAL AREAS OF SUPERIOR


TEMPORAL GYRUS.
1. Primary auditory area (41,42)
2. Secondary auditory area
3. Sensory speech area (Wernicke’s area)

Page 204 of 284


NAME THE TWO SENSORY THALAMIC NUCLEI
1. Medial geniculate body
2. Lateral geniculate body

 These are located posteroventral to the pulvinar.


 Conventionally these nuclei are described under metathalamus,
but nowadays they are considered as thalamic nuclei.

INSULA
 The insula is the submerged (hidden) portion of the cerebral cortex in
the floor of the lateral sulcus.
 It is triangular in shape and surrounded all around by a sulcus, the
circular sulcus except anteroinferiorly at its apex called limen
insulae which is continuous with the anterior perforated substance.
 The insula is divided into two regions – anterior and posterior by a
central sulcus.
 The anterior region presents 3 or 4 short gyri called gyri brevis and
the posterior region presents 1 or 2 long gyri called gyri longa.
1. The insula is hidden from the surface view by the overgrown
cortical areas of frontal, parietal and lobes.
2. These areas are termed frontal, frontoparietal and temporal
opercula (operculum = lid).
3. The superior surface of the temporal operculum presents anterior
and posterior transverse temporal gyri.

 The middle cerebral artery and deep middle cerebral vein lie on the
surface of the insula.

Page 205 of 284


Page 206 of 284
SUPEROLATERAL SURFACE OF CEREBRUM
INTRODUCTION:
 Most convex and most extensive surface of cerebrum.
 Faces upwards and laterally.

IMPORTANT SULCI:
1. Central sulci {of Rolando}
2. Lateral sulci {of sylvius}

CENTRAL SULCI OF ROLANDO:


1. Begins at superomedial border of the hemisphere, 1cm behind the
midpoint between frontal and occipital pole.
2. Runs on superolateral surface, obliquely downwards and forwards
3. Terminates just above the posterior ramus of lateral sulcus

Page 207 of 284


LATERAL SULCI OF SYLVIUS:
PARTS –
1. STEM
2. RAMI

STEM:
1. Begins as a deep cleft on the inferior surface of the cerebral
hemisphere
2. Extends laterally to reach superolateral surface and divides into 3
Rami

RAMI:
1. Anterior horizontal
2. Anterior ascending
3. Posterior

SULCI AND GYRI IN DIFFERENT LOBES OF


CERBRUM:
FRONTAL LOBE:

1. SULCI –
1. Pre central
2. Superior frontal
3. Inferior frontal
4. Anterior horizontal and
5. Anterior ascending rami of lateral sulcus

2. GYRI -
1. Pre central
2. Superior frontal
3. Middle frontal
4. Inferior frontal

Page 208 of 284


 PRE-CENTRAL SULCUS runs parallel to and infront of the
central sulcus, the pre central gyrus lies between the two sulci
 SUPERIOR and INFERIOR FRONTAL SULCI divided the rest of
the areas of frontal lobe into superior, middle and inferior frontal
gyri
 ANTERIOR HORIZONTAL and ANTERIOR ASCENDING
RAMI OF LATERAL SULCUS subdivides the inferior frontal
gyrus into pars orbitalis, pars triangularis and pars opercularis

PARIETAL LOBE:
1. SULCI
1. Post central
2. Intraparietal

2. GYRI
1. Post central
2. Superior parietal lobule
3. Inferior parietal lobule
 POST CENTRAL SULCUS runs parallel to and behind the central
sulcus, the post central gyrus lies between the two sulci
 The area behind the post central gyrus is divided into the superior
and inferior parietal lobule by the INTRAPARIETAL SULCUS
 Upturned ends of POSTERIOR RAMUS OF LATERAL SULCUS
and superior and inferior temporal sulci divides the Inferior
parietal lobule into Anterior part (Supra marginal Gyrus), middle
part (angular gyrus) and posterior part.

3. TEMPORAL LOBE
Superior and inferior temporal sulci divide the temporal lobe
into superior, middle and inferior temporal Gyri

Page 209 of 284


4. OCCIPITAL LOBE
1. LATERAL OCCIPITAL SULCUS divided it into superior
and inferior occipital gyri
2. LUNATE SULCUS separates these gyri from occipital pole
3. TRANSVERSE OCCIPITAL SULCUS separates the arcus
parieto-occipitalis from superior occipital gyrus

Page 210 of 284


BLOOD SUPPLY OF THE SPINAL CORD

ARTERIAL SUPPLY:
The arterial supply for the spinal cord by-
1. Anterior spinal artery.
2. Two posterior spinal arteries.
3. Segmental arteries.
4. Branches from arterial plexus (arteria vasocorona) lying on pia
mater.

Page 211 of 284


ANTERIOR SPINAL ARTERY
 Formed by the union of two small spinal branches of the right and
left vertebral arteries in the upper cervical canal.
 It runs caudally in the anterior median fissure of the spinal cord
 Terminates along the filum terminale.
 Supply - Anterior 2/3rd of Spinal cord

POSTERIOR SPINAL ARTERY


 There are two posterior spinal arteries.
 Each artery arises as a small branch from either the vertebral or
posterior inferior cerebellar artery.
 Each posterior spinal artery runs down on the posterolateral
aspect of the cord in the posterolateral sulcus along the line of
attachment of posterior nerve roots
 Posterior spinal artery divides into two collateral arteries along
the medial and lateral sides of the posterior nerve roots. Thus,
there are five longitudinal arteries around the spinal cord.
 Supply - posterior 1/3rd of spinal cord

SEGMENTAL ARTERIES
 They are spinal branches of:
1. Deep cervical,
2. Ascending cervical,
3. Posterior intercostal,
4. Lumbar and
5. Lateral sacral arteries.
 They reach the spinal cord, as the anterior and posterior radicular
arteries along the corresponding roots of the spinal nerves.
 FUNCTION - nourish the nerve roots.
 There are about 8 anterior and 12 posterior radicular arteries which
reach the spinal cord.

Page 212 of 284


 (One of the anterior radicular branch is very large and is called the
Anterior Radicularis Magna)

VENOUS DRAINAGE:
Veins are arranged in the form of 6 longitudinal channels:
1. TWO MEDIAN LONGITUDINAL - one in the anterior median
fissure and the other in the posteromedian sulcus
2. TWO ANTEROLATERAL - one on either side, posterior to the
anterior nerve roots.
3. TWO POSTEROLATERAL - one on either side, posterior to the
posterior nerve roots.

COMMUNICATIONS:
 These longitudinal venous channels communicate with the internal
vertebral venous plexus.

Page 213 of 284


CIRCLE OF WILLIS (CIRCULUS ARTERIOSUS)
 Hexagonal arterial circle around the interpeduncular fossa at the
base of the brain.

LOCATION:
 Interpeduncular sub-arachnoid cistern

FORMATION:
 Branches of basilar and internal carotid artery.

1. ANTERIORLY
 Anterior communicating artery
 Anterior cerebral artery
2. POSTERIORLY
 Right and left posterior cerebral arteries
3. LATERALLY
 Posterior communicating artery which connects the internal
carotid artery with posterior cerebral artery

BASILAR SYSTEM:
Branches of basilar artery-

PONTINE BRANCHES
 There are numerous short slender paramedian vessels which
pierce the pons to supply it.
ANTERIOR INFERIOR CEREBELLAR ARTERY
 They arise close to the lower border of the pons and runs
backwards and laterally usually ventral to the VII and VIII
cranial nerves.
 Then it forms a loop over the flocculus of the cerebellum and
peeps into the internal acoustic meatus for a variable distance
lying below the and VII cranial nerves. After exit from the

Page 214 of 284


meatus, it supplies the anterolateral portion of the inferior
surface of the cerebellum.
LABYRINTHINE ARTERY
 It is a long slender branch which arises either from basilar artery
or from anterior inferior cerebellar artery.
 It accompanies the vestibulocochlear nerve and enters the internal
auditory meatus to supply the internal ear. It is an end artery.

Page 215 of 284


SUPERIOR CEREBELLAR ARTERY
 It arises close to the superior border of the pons, runs laterally
below the oculomotor nerve (which is interposed between this
artery and the posterior cerebral artery), and winds round the
cerebral peduncle below the trochlear nerve to reach the superior
surface of the cerebellum which it supplies.

POSTERIOR CEREBRAL ARTERY


 It passes laterally parallel to the superior cerebellar artery, curves
around the midbrain to reach the medial surface of the cerebral
hemisphere, beneath the splenium of corpus callosum.
 The artery gives off temporal branches which ramify over the
inferior surface of the temporal lobe, and calcarine and parieto-
occipital branches which run along the corresponding sulci.

VERTEBRAL SYSTEM:

ANTERIOR SPINAL ARTERY


 It is a small branch arising near the termination of the vertebral
artery. It descends in front of the medulla and unites with its
fellow of the opposite side at the level of the lower end of the olive
to form a single median trunk that descends along the anterior
longitudinal fissure of the spinal cord.

POSTERIOR SPINAL ARTERY


 It arises from vertebral artery and sometimes from posterior
inferior cerebellar artery. It passes downwards on the posterior
surface of the spinal cord, after dividing into two branches; one
along the medial side, and the other along the lateral side of the
dorsal roots of the spinal nerves.

Page 216 of 284


POSTERIOR INFERIOR CEREBELLAR ARTERY
 It is the largest branch of the cranial (4th) part of the vertebral
artery. It arises near the lower end of the olive, winds backwards
around the medulla oblongata, and then ascends to the
pontomedullary junction.
MENINGEAL BRANCHES are small and supply the dura mater of
the posterior cranial fossa.

MEDULLARY ARTERIES are several minute vessels which supply


the medulla oblongata.

CAROTID SYSTEM: Branches of carotid arteries

ANTERIOR CEREBRAL ARTERY


 It is a smaller terminal branch of the internal carotid artery.
 It runs forwards and medially above the optic nerve to the
commencement of the median longitudinal cerebral fissure, where
it comes very close to its fellow of the opposite side and gets joined
with it by a short transverse anterior communicating artery.
 The anterior cerebral artery then curves around the genu of
corpus callosum.
 The branches given off just distal to the anterior communicating
artery supply the medial part of the orbital surface of the frontal
lobe.

MIDDLE CEREBRAL ARTERY


 Branch of internal carotid artery that is given laterally.
 runs laterally in the stem of the lateral sulcus
 Then, turns backwards and upwards in the posterior ramus of the
lateral sulcus, where it breaks up into frontal, parietal and
temporal branches which emerge from the lateral sulcus and run
towards the areas of their supply.

Page 217 of 284


POSTERIOR COMMUNICATING ARTERY
 Arises close to the termination of the internal carotid artery.
 It runs backwards and anastomoses with the proximal part of
the posterior cerebral artery.

VENOUS DRAINAGE OF BRAIN:


 The characteristics of the veins of the brain are

1. The walls are devoid of muscle


2. The veins have no valves
3. To maintain patency some of them open in to the cranial, venous
sinuses against the direction of the blood flow in the sinus.
 E.g., the superior cerebral veins draining into the superior sagittal
sinus.
 The group of veins that drain blood from the brain are

EXTERNAL CEREBRAL VEINS:


1. SUPERIOR CEREBRAL VEINS
 6 to 12 in number
 Drain superolateral surface of hemisphere
 Terminate in superior sagittal sinus
2. SUPERFICIAL MIDDLE CEREBRAL VEIN
 Drains the area around the posterior remains of the lateral
sulcus
 Terminus into cavernous sinus
 It communicates with superior sagittal and transverse
sinuses and also with deep middle cerebral vein
3. DEEP MIDDLE CEREBRAL VEIN
 Drains the surface of insular and terminates in basal vein
4. INFERIOR CEREBRAL VEIN
 Divided into orbital and tentorial veins

Page 218 of 284


 Orbital veins terminate in the superior sagittal sinus
 Tentorial veins terminate in the cavernous or any other
surrounding sinuses
5. ANTERIOR CEREBRAL VEIN
 Drain the Corpus callosum and the anterior part of the
medial surface of the hemisphere
 Terminate in the basal vein

INTERNAL CEREBRAL VEINS:


 One vein on each side
 Formed by the union of thalamostriate and choroidal veins at the
apex of the Tela choroidea of third ventricle
 The right and the left vein in the Tela choroidea unite to form
great cerebral vein below the splenium of the Corpus callosum

TERMINAL VEINS:
1. GREAT CEREBRAL VEIN OF GALEN
 It is a single median vein formed by the union of two
internal cerebral veins.
 Its tributaries are basal veins and veins from pineal body,
the colliculi, the cerebellum and the joining adjoining part of
the occipital lobes of cerebrum
2. BASAL VEIN
 One on each side
 Form at the anterior perforated substance by the union of
the deep middle cerebral vein the anterior cerebral veins and
striate vein
 It terminates by joining the great cerebral vein

Page 219 of 284


 Its tributaries include small veins from cerebral peduncle,
interpeduncular structures, the tectum of midbrain and the
Parahippocampal gyrus
 Ultimately all veins drain into the various cranial venous
sinuses which in turn drain into the internal jugular vein.

BLOOD SUPPLY OF BRAINSTEM


1. MIDBRAIN
 Supplied by branches from posterior cerebral arteries
including the central branches both posteromedial and
posterolateral.
2. PONS
 Pontine branches of basilar artery
3. MEDULLA
 Medullary branches of vertebral artery
 Branches from posterior inferior cerebral artery.
 Veins of the brainstem drain into neighbouring venous
sinuses.

Page 220 of 284


BLOOD SUPPLY OF BRAIN

ARTERIAL SUPPLY:
 By circle of Willis and its branches
 Circle of Willis (circulus arteriosus)
 Hexagonal arterial circle around the interpeduncular fossa at the
base of the brain.

LOCATION:
 Interpeduncular sub-arachnoid cistern

Page 221 of 284


FORMATION:
 Branches of basilar and internal carotid artery.

1. ANTERIORLY
 Anterior communicating artery
 Anterior cerebral artery
2. POSTERIORLY
 Right and left posterior cerebral arteries
3. LATERALLY
 Posterior communicating artery which connects the internal
carotid artery with posterior cerebral artery

Page 222 of 284


BASILAR SYSTEM: BRANCHES OF BASILAR ARTERY-
PONTINE BRANCHES
 They are numerous short slender paramedian vessels which
pierce the pons to supply it.

ANTERIOR INFERIOR CEREBELLAR ARTERY


 They arise close to the lower border of the pons and runs
backwards and laterally usually ventral to the VII and VIII
cranial nerves.
 Then it forms a loop over the flocculus of the cerebellum and
peeps into the internal acoustic meatus for a variable distance
lying below the VII and VIII cranial nerves. After exit from the
meatus, it supplies the anterolateral portion of the inferior
surface of the cerebellum.

LABYRINTHINE ARTERY
 It is a long slender branch which arises either from basilar artery
or from anterior inferior cerebellar artery.
 It accompanies the vestibulocochlear nerve and enters the internal
auditory meatus to supply the internal ear. It is an end artery.

SUPERIOR CEREBELLAR ARTERY


 It arises close to the superior border of the pons, runs laterally
below the oculomotor nerve (which is interposed between this
artery and the posterior cerebral artery), and winds round the
cerebral peduncle below the trochlear nerve to reach the superior
surface of the cerebellum which it supplies.

POSTERIOR CEREBRAL ARTERY


 It passes laterally parallel to the superior cerebellar artery, curves
around the midbrain to reach the medial surface of the cerebral
hemisphere, beneath the splenium of corpus callosum.

Page 223 of 284


 The artery gives off temporal branches which ramify over the
inferior surface of the temporal lobe, and calcarine and parieto-
occipital branches which run along the corresponding sulci.

VERTEBRAL SYSTEM:
ANTERIOR SPINAL ARTERY
 It is a small branch arising near the termination of the vertebral
artery.
 It descends in front of the medulla and unites with its fellow of the
opposite side at the level of the lower end of the olive to form a
single median trunk that descends along the anterior longitudinal
fissure of the spinal cord.

POSTERIOR SPINAL ARTERY


 It arises from vertebral artery and sometimes from posterior
inferior cerebellar artery.
 It passes downwards on the posterior surface of the spinal cord,
after dividing into two branches; one along the medial side, and
the other along the lateral side of the dorsal roots of the spinal
nerves.

POSTERIOR INFERIOR CEREBELLAR ARTERY


 It is the largest branch of the cranial (4th) part of the vertebral
artery.
 It arises near the lower end of the olive, winds backwards around
the medulla oblongata, and then ascends to the pontomedullary
junction.

MENINGEAL BRANCHES are small and supply the dura mater of


the posterior cranial fossa.
MEDULLARY ARTERIES are several minute vessels which supply
the medulla oblongata.

Page 224 of 284


CAROTID SYSTEM: Branches of carotid arteries
ANTERIOR CEREBRAL ARTERY
 It is a smaller terminal branch of the internal carotid artery.
 It runs forwards and medially above the optic nerve to the
commencement of the median longitudinal cerebral fissure, where
it comes very close to its fellow of the opposite side and gets joined
with it by a short transverse anterior communicating artery.
 The anterior cerebral artery then curves around the genu of
corpus callosum.
 The branches given off just distal to the anterior communicating
artery supply the medial part of the orbital surface of the frontal
lobe

MIDDLE CEREBRAL ARTERY


 Branch of internal carotid artery that is given laterally.
 runs laterally in the stem of the lateral sulcus
 Then, turns backwards and upwards in the posterior ramus of the
lateral sulcus, where it breaks up into frontal, parietal and
temporal branches which emerge from the lateral sulcus and run
towards the areas of their supply.

POSTERIOR COMMUNICATING ARTERY


 Arises close to the termination of the internal carotid artery.
 It runs backwards and anastomoses with the proximal part of the
posterior cerebral artery.

Page 225 of 284


VENOUS DRAINAGE OF BRAIN:

 The characteristics of the veins of the brain are


 The walls are devoid of muscle
 The veins have no valves
 To maintain patency some of them open in to the cranial, venous
sinuses against the direction of the blood flow in the sinus.
 E.g., the superior cerebral veins draining into the superior sagittal
sinus.
 The group of veins that drain blood from the brain are

EXTERNAL CEREBRAL VEINS:


SUPERIOR CEREBRAL VEINS
 6 to 12 in number
 Drain superolateral surface of hemisphere
 Terminate in superior sagittal sinus

Page 226 of 284


SUPERFICIAL MIDDLE CEREBRAL VEIN
 Drains the area around the posterior remains of the lateral sulcus
 Terminates into cavernous sinus
 It communicates with superior sagittal and transverse sinuses
and also with deep middle cerebral vein

DEEP MIDDLE CEREBRAL VEIN


 Drains the surface of insular and terminates in basal vein

INFERIOR CEREBRAL VEIN


 Divided into orbital and tentorial veins
 Orbital veins terminate in the superior sagittal sinus
 Tentorial veins terminate in the cavernous or any other
surrounding sinuses

ANTERIOR CEREBRAL VEIN


 Drain the Corpus callosum and the anterior part of the medial
surface of the hemisphere
 Terminate in the basal vein

INTERNAL CEREBRAL VEINS:


 One vein on each side
 Formed by the union of thalamostriate and choroidal veins at the
apex of the Tela choroidea of third ventricle
 The right and the left vein in the Tela choroidea unite to form
great cerebral vein below the splenium of the Corpus callosum

Page 227 of 284


TERMINAL VEINS:
 Great cerebral vein of Galen
 It is a single median vein formed by the union of two internal
cerebral veins.
 Its tributaries are basal veins and veins from pineal body, the
colliculi, the cerebellum and the joining adjoining part of the
occipital lobes of cerebrum

BASAL VEIN
 One on each side
 Form at the anterior perforated substance by the union of the
deep middle cerebral vein the anterior cerebral veins and striate
vein
 It terminates by joining the great cerebral vein
 Its tributaries include small veins from cerebral peduncle,
interpeduncular structures, the tectum of midbrain and the
Parahippocampal gyrus
 Ultimately all veins drain into the various cranial venous sinuses
which in turn drain into the internal jugular vein.

BLOOD SUPPLY OF BRAINSTEM


MIDBRAIN
 supplied by branches from posterior cerebral arteries including
the central branches both posteromedial and posterolateral.

PONS
 Pontine branches of basilar artery

MEDULLA
 Medullary branches of vertebral artery
 Branches from posterior inferior cerebral artery.
 Veins of the brainstem drain into neighbouring venous sinuses.

Page 228 of 284


NAME OF THE BONES MEETING AT PTERION
 The PTERION is the H-shaped formation of sutures on the side of
the calvarium representing the junction of four skull bones:

1. The greater wing of sphenoid bone


2. Squamous portion of the temporal bone
3. Parietal bone
4. Frontal bone

Page 229 of 284


FALX CEREBRI
 The falx cerebri is a large sickle shaped fold of dura mater occupying
the median longitudinal fissure between the two cerebral
hemispheres.

 It has two ends:


1. The anterior end is narrow and is attached to the crista galli.
2. The posterior end is broad and is attached along the median
plane to the upper surface of the tentorium cerebelli.

Page 230 of 284


The falx cerebri has two margins:

1. The upper margin is convex and is attached to the lips of the


sagittal sulcus.
2. The lower margin is concave and free.

 The falx cerebri has right and left surfaces each of which is related to
the medial surface of the corresponding cerebral hemisphere.
 Three important venous sinuses are present in relation to this fold.
 The superior sagittal sinus lies along the upper margin, the inferior
sagittal sinus along the lower margin, and the straight sinus along
the of attachment of the falx to the tentorium cerebelli.

Page 231 of 284


EPICRANIAL APONEUROSIS
 The third layer of scalp is the deep fascia in the form of the
epicranial aponeurosis or gale aponeurotica with the
occipitofrontalis muscle.
 The epicranial aponeurosis is freely movable on the pericranium
along with the overlying and adherent skin and fascia.
 Anteriorly it receives the insertion of the frontalis and posteriorly
it receives the insertion of the occipitalis and is attached to the
external occipital protuberance, and to the highest nuchal lines
in between the occipital bellies.
 On each side, the aponeurosis is attached to the superior
temporal line, but sends down a thin expansion which passes
over the temporal fascia and is attached to the zygomatic arch.

SHOW THE VENOUS SINUSES RELATED TO FALX CEREBRI

Three important venous sinuses are present in relation to this fold. The
superior sagittal sinus lies along the upper margin, the inferior sagittal
sinus along the lower margin, and the straight sinus along the of
attachment of the falx to the tentorium cerebelli.

NAME THE STRUCTURES PASSING THROUGH INTERNAL


ACOUSTIC MEATUS

 Internal acoustic meatus lies in the temporal bone and exists


between the inner ear and posterior cranial fossa.
 It includes the vestibulocochlear nerve (CN VIII), facial nerve (CN
VII), the labyrinthine artery, and the vestibular ganglion.

Page 232 of 284


NAME THE STRUCTURE PASSING THROUGH FORAMEN
SPINOSUM

 The foramen spinosum is located in the posteromedial part of


greater wing of sphenoid bone posterolateral to foramen ovale
which connects the middle cranial fossa with the infratemporal
fossa.
 It transmits the middle meningeal artery, middle meningeal vein,
and the nervus spinosus.

NAME THE STRUCTURES PRESENT IN LATERAL WALL OF


CAVERNOUS SINUS

Structures within the lateral wall from above downwards:

1. OCULOMOTOR NERVE: in the anterior part of the sinus, it


divides into superior and inferior divisions which leave the
sinus by passing through the superior orbital fissure.
2. TROCHLEAR NERVE: in the anterior part of the sinus, it
crosses superficial to the oculomotor nerve, and enters the orbit
through the superior orbital fissure.
3. OPHTHALMIC NERVE: in the anterior part of the sinus, it
divides into the lacrimal, frontal and nasociliary nerves.
4. MAXILLARY NERVE: it leaves the sinus by passing through
the foramen rotundum on its way to the pterygopalatine fossa.
5. TRIGEMINAL GANGLION: the ganglion and its Dural cave
may project into the lateral wall of the sinus.

Page 233 of 284


Page 234 of 284
PTERION
 The pterion is the H-shaped formation of sutures on the side of the
calvarium representing the junction of four skull bones:
1. The greater wing of the sphenoid bone
2. Squamous portion of the temporal bone
3. Frontal bone
4. Parietal bone

 Pterion site in the anterolateral fontanelle is the thin part of skull.


In accidents, the anterior division of middle meningeal artery at
pterion may be ruptured, leading to clot formation between the skull
and dura mater or extradural haemorrhage.
 The clot compresses the motor area of the brain and causes paralysis
in the opposite side of the body.
 The clot must be sucked out at the earliest.

EMISSARY VEIN

1. Emissary veins connect the extra cranial veins with the


intracranial veins.
2. venous sinuses to equalise the pressure.

Page 235 of 284


VEINS
FORAMEN OF VENOUS
NAME OUTSIDE
SKULL SINUS
SKULL

Parietal venous Superior


Parietal foramen Veins of scalp
sinus sagittal sinus

Mastoid venous
Mastoid foramen Veins of scalp Sigmoid sinus
sinus

Internal jugular
Emissary vein Hypoglossal canal Sigmoid sinus
vein

Condylar emissary Posterior condylar Suboccipital


Sigmoid sinus
vein foramen venous plexus

Pharyngeal Cavernous
2- 3 emissary vein Foramen lacerum
venous plexus sinus

Pterygoid venous Cavernous


Emissary vein Foramen ovale
plexus sinus

Veins from the


Superior
Emissary vein Foramen caecum upper part of
sagittal sinus
nose

Page 236 of 284


LIST OUT THE PAIRED DURAL VENOUS
SINUSES
 There are 8 paired Dural venous sinus:

1. Cavernous sinus

2. Superior petrosal sinus

3. Inferior petrosal sinus

4. Transverse sinus

5. Sigmoid sinus

6. Sphenoparietal sinus

7. Petrosquamous sinus

8. Middle meningeal sinus

Page 237 of 284


LIST OUT THE UNPAIRED DURAL VENOUS
SINUSES
 There are 7 unpaired Dural venous sinus:

1. Superior sagittal sinus

2. Inferior sagittal sinus

3. Straight sinus

4. Occipital sinus

5. Anterior intercavernous sinus

6. Posterior intercavernous sinus

7. Basilar plexus of veins

Page 238 of 284


TENTORIUM CEREBELLI
 It is a tent-shaped fold of the dura mater forming the roof of the
posterior cranial fossa.
 It separates the cerebellum from the occipital lobes of the
cerebrum.
 It has two margins and two surfaces.

MARGINS

 The inner free margin is U-shaped and encloses the tentorial


notch for the passage of the midbrain. The anterior ends of the
concave free margin are attached to the anterior clinoid processes.
 The outer attached margin is convex and attached on each side
(from before backward) to the posterior clinoid process, the
posteroinferior angle of the parietal bone, and the lips of
transverse sulci on the occipital bone.
 The free and attached margins cross each other near the apex of
the petrous temporal bone to enclose a triangular area anteriorly
which is pierced by the oculomotor nerve.

SURFACES

 The convex upper surface slopes to either side from the median
plane. In the median plane, it provides attachment to falx cerebri.
 The concave inferior surface provides attachment to falx cerebelli
in its posterior part.

TRIGEMINAL OR MECKEL’S CAVE:

 It is a recess of the dura mater present in relation to the attached


outer margin of the tentorium cerebelli.

Page 239 of 284


 It is formed by the evagination of the meningeal layer of the dura
mater by two roots of the trigeminal nerve below the superior
petrosal sinus.

VENOUS SINUS ENCLOSED IN THE TENTORIUM


CEBRI:

1. TRANSVERSE SINUS within the posterior part of the attached


margin.

2. SUPERIOR PETROSAL SINUS within the anterolateral part of


attached margin.

3. STRAIGHT SINUS within in the line of attachment between the


falx cerebri and tentorium cerbelli.

Page 240 of 284


SUBOCCIPITAL NERVE
 The suboccipital nerve is the dorsal primary ramus of the first
cervical nerve (C1). It exits the spinal cord between the skull and the
first cervical vertebra, the atlas. It lies within the suboccipital
triangle along with the vertebral artery.
 It supplies the four suboccipital muscles and the semispinalis capitis.
 The nerve to the inferior oblique gives off communicating ranch to
the greater occipital nerve capitis.

TRIBUTARIES OF CAVERNOUS SINUS


 The tributaries or incoming channels are as follows:

FROM THE ORBIT:

1. The superior ophthalmic vein


2. A branch of the inferior ophthalmic vein
3. The central vein of retina

FROM THE BRAIN:

1. Superior middle cerebral vein


2. Inferior cerebral vein from the temporal lobe

FROM THE MENINGES:

1. Sphenoparietal sinus
2. The frontal trunk of the middle meningeal vein may drain
directly into the pterygoid plexus through the foramen ovale.

Page 241 of 284


STRUCTURES PASSING THROUGH THE
FORAMEN OVALE
 Foramen ovale transmits the
following structures:
1. The mandibular nerve
2. The accessory meningeal
artery
3. The lesser petrosal nerve
4. An emissary vein
connecting cavernous sinus
with the pterygoid plexus of
veins.
5. Anterior trunk of middle
meningeal artery

LIGAMENTUM DENTICULATUM
 Ligamentum denticulatum are 21 pairs of teeth like projections.
 They fuse laterally with the arachnoid and duramater midway
between the exits of the roots of adjacent spinal nerves.
 The highest process attaches immediately superior to the foramen
magnum.
 The ligamentum denticulate keeps the spinal cord in position.

Page 242 of 284


ARACHNOID GRANULATIONS
 Arachnoid granulations are structures filled with cerebrospinal
fluid (CSF) that extend into the venous sinuses through
openings in the dura mater and allow the drainage of CSF from
subarachnoid space into venous system.
 The lesions are primarily located in the parasagittal region
along the superior sagittal sinus, which is occasionally seen at
the transverse sinus.

CROSS SECTIONS OF SPINAL CORD AT CERVICAL LEVEL

 At the level of cervical vertebrae, the segment of spinal cord is oval


shaped
1. POSTERIOR HORN - slender
2. LATERAL HORN - absent
3. ANTERIOR HORN - narrow in 1-3 segments, broad in C4 -C8
segments to supply the upper limbs.

Page 243 of 284


CROSS SECTION OF SPINAL CORD AT
THORACIC LEVEL
 At the level of thoracic level, the segment of spinal cord is circular.
 Posterior horn- slender
 Lateral horn- present for thoracolumbar outflow
 Anterior horn- slender in T2- T12 segments, broad in T1 segment.

Page 244 of 284


LUMBAR PUNCTURE
 The lumbar puncture is done to draw CSF from subarachnoid space
for diagnostic and therapeutic purposes.
 The needle is inserted between L3 and L4 or L4 and L5 vertebrae
with patient’s back flexed either in sitting position or lying on a bed
in left lateral position, usually the later when patient is curled up
lying on left side.
 The needle passes through the supraspinous and interspinous
ligaments and between ligamenta flava before piercing the dura
mater. As the dura mater is pierced, there is a distinct feel of give
way.
 Since the spinal cord ends at the lower border of L1 vertebra, there is
no danger of damage to the spinal cord

NERVES RISING FROM MEDULLA OBLONGATA


 Rootlets of the hypoglossal nerve: These emerge from the
anterolateral sulcus between the pyramid and the olive.
 Rootlets of the 9th, 10th, and 11th (cranial part) cranial nerves:
These emerge through the posterolateral sulcus separating the olive
from the inferior cerebellar peduncle.

Page 245 of 284


LIST SOMATIC AFFERENT NUCLEI
 General somatic afferent (GSA) fibres.
 They carry general sensations of pain, touch, and temperature
from skin and proprioceptive sensations of vibration and muscle
and joint sense.

MODIFICATIONS OF CRANIAL PIA MATTER


 The pia mater provides sheath for the cranial nerves merging with
the epineurium around them.
 It also provides perivascular sheath for the minute vessels
entering around them.
 Folds of pia mater enclosing the tufts of capillaries form the
telanchoroidea. Such pia mater lined by secretary ependyma from
the choroidal plexus.

TERMINAL BRANCHES OF FACIAL NERVE


● Five terminal branches
1. Temporal
2. Zygomatic
3. Buccal – upper
buccal and lower
buccal nerves
4. Marginal mandibular
nerve
5. Cervical nerve

Page 246 of 284


INDICATE THE PALEO CEREBELLAR DEEP
NUCLEI
 Globose nucleus and Emboliform nucleus are the paleo cerebellar
nuclei
 They are collectively called as Nucleus interpositus

DEEP NUCLEI OF CEREBELLUM (ALSO CALLED


AS INTRACEBELLAR NUCLEI OR CENTRAL
NUCLEI)
 From lateral to medial
1. DENTATE NUCLEUS - Nucleus of neocerebellum; largest
nucleus
2. EMBOLIFORM NUCLEUS - Nucleus of paleocerebellum
3. GLOBOSE NUCLEUS - Nucleus of paleocerebellum
4. FASTIGIAL NUCLEUS - Nucleus of archicerebellum

LIST SPECIAL SOMATIC AFFERENT NUCLEI


1. COCHLEAR NUCLEI - Dorsal, ventral → Receives fibres from
spiral ganglion
2. VESTIBULAR NUCLEI - Medial, lateral, inferior, superior
→Receives fibres from cerebellum

Page 247 of 284


MIDDLE MENINGEAL ARTERY
 Branch of first part of Maxillary artery.
 Passes through foramen spinosum, enters cranial cavity, supply
meninges.
 Divides into anterior and posterior branches in cranial cavity.
 Anterior branch crosses pterion and posterior branch ascends
backwards towards lambda.
 In skull bone fracture or head injury this artery may get ruptured
deep to pterion leading to EXTRADURAL HEMATOMA.

PINEAL GLAND
 Neuroendocrine gland secreting melatonin.
 Situated in a vertical groove between 2 superior colliculi below
splenium of corpus callosum.
 The stalk has two laminae-
1. SUPERIOR -continues with habenular commissure.
2. INFERIOR-continues with posterior commissure.
 Calcium phosphates and carbonates gets deposited in pineal gland
with age from of multilaminar corpuscles -called BRAIN SAND OR
CORPORA ARENACEA.
 Pineal gland is supplied by nervus conarii.
 It acts as a biological clock.
 Pineal gland is the only part of brain, which has no nerve cells in it.

Page 248 of 284


ORIGIN AND BRANCHES OF MIDDLE
MENINGEAL ARTERY
ORIGIN:

 Largest meningeal branch -first part of Maxillary artery.


 Ascends up into cranial cavity via foramen spinosum.

BRANCHES: Anterior frontal branch-

 Courses up towards pterion, then curves downward ascend towards


cortex.
 In pterion region, middle meningeal artery frequently lies on bony
tunnel in parietal bone.

POSTERIOR PARIETAL BRANCH-

 Arch backwards on squamous part of temporal bone.


 Crosses lower border of parietal bone in front of mastoid angle.
 Gives branches that supplies posterior part of dura mater and
cranium.

Page 249 of 284


FORMATION OF BASAL VEIN
 Formed at base of brain in the region of anterior perforated
substance by union of 3 veins
1. ANTERIOR CEREBRAL VEIN
2. DEEP MIDDLE CEREBRAL VEIN
3. STRIATE VEINS-from anterior perforated substance
 Runs posteriorly around midbrain, medial to uncus,
parahippocampus.
 Terminates in Greater cerebral vein (vein of Galen) below
splenium of corpus callosum.
 Basal vein is also called “VEIN OF ROSENTHAL”.

BASILAR ARTERY
 Formed by union of 2 vertebral arteries at the lower border of pons.
 Basilar artery ascends in midline on ventral surface of pons and at
its upper border terminates into right and left posterior cerebral
arteries

BRANCHES:

1. Pontine branches
2. Anterior inferior Cerebellar arteries
3. Labyrinthine artery
4. Superior Cerebellar artery
5. Posterior cerebral arteries

Page 250 of 284


CORPUS STRIATUM
 Located lateral to thalamus.
 Part of basal ganglia.
 Includes caudate nucleus and lentiform nucleus.

STRUCTURES FORMING LIMBIC SYSTEM

CORTICAL STRUCTURES-

1. LIMBIC LOBE- Cingulate gyrus, isthmus, Parahippocampal


gyrus, uncus
2. HIPPOCAMPAL FORMATION- hippocampus, dentate
gyrus, gyrus fasciolaris, indusium griseum.

Page 251 of 284


SUBCORTICAL NUCLEI

1. Amygdaloid body
2. Septal region and nuclei
3. Olfactory areas
4. Mammillary bodies
5. Anterior nucleus of thalamus

FIBRE BUNDLES

1. Fornix
2. Mammillothalamic trac
3. Stria terminalis
4. Anterior commissure
5. Stria medullaris thalami
6. Median forebrain bundle
7. Cingulum
8. Diagonal band of Broca

COMPONENTS OF BASAL GANGLIA


1. Corpus striatum (caudate nucleus and lentiform nucleus)
2. Claustrum
3. Amygdaloid body
4. Substantia nigra, red nucleus
5. Subthalamic nucleus

Page 252 of 284


PARKINSONISM
 Also called Parkinson’s disease or paralysis agitans
 Due to deficiency of dopamine in corpus striatum
 Due to lesion in Substantia nigra or nigrostriatal fibres

FEATURES:

1. Resting tremors
2. Lead pipe muscular rigidity
3. Mask-like face (no facial expression)
4. Shuffling gait
5. Pill rolling movements of hand
6. Absence of associated movements

FUNCTIONAL AREAS OF SUPERIOR


TEMPORAL GYRUS
 PRIMARY AUDITORY AREA (Brodmann’s area 41 and 42) located
in superior surface of superior temporal gyrus occupying anterior
transverse temporal gyrus (Heschl's gyrus)
 SECONDARY AUDITORY AREA (Brodmann’s area 22) lateral
surface of superior temporal gyrus posterior to primary auditory
area.
 Receives fibres from medial geniculate body via auditory radiation.
 Cochlea is bilaterally represented.
 Lesion in one cortex does not cause unilateral deafness.

Page 253 of 284


SENSORY THALAMIC NUCLEI
 Medial geniculate body -auditory
 Lateral geniculate body-visual
 Ventral posterior nucleus -Somatosensory, vestibular

INSULA
 Insula is the submerged /hidden portion of cerebral cortex in floor of
lateral sulcus
 Triangle in shape surrounded by sulcus, the circular sulcus except at
the apex.
 It can be seen only when lips of lateral sulcus are widely pulled apart
 During brain development, due to overgrowth of surrounding cortical
areas it is submerged
 Also called ISLAND OF REAL or CENTRAL LOBE or FIFTH
LOBE
 It’s is divided into anterior and posterior region by Central sulcus.
 Anterior region presents 3 or 4 short gyri called gyri brevia and
posterior region presents long gyri called gyri longa.

VISUAL STRIA
 It is also called Line of Gennari, it refers to the primary visual
cortex, Broadman's area No-17, located in the occipital Lobe.

Page 254 of 284


STRUCTURES LODGED IN LATERAL SULCUS
OF CEREBRUM
1. Superficial Middle cerebral vein

2. Left lateral cerebral artery with its superficial and inferior


terminal branches.

PARTS OF CAUDATE NUCLEUS


1. Head-Rounded anterior part in front of interventricular foramen

2. Body

3. Tail-merges at its anterior extremity with an almond shaped mass


of grey matter-amygdaloid body

CORPUS CALLOSUM
 It is the largest of the commissural fibres (interhemispheric
fibres).

 It consists of about 100 million fibres.

 It is largely responsible for interhemispheric transfer of


information which is essential for bilateral responses and learning
processes.

Page 255 of 284


PARTS OF CORPUS CALLOSUM
1. Rostrum (Anterior)

2. Genu

3. Body/Trunk (Middle part)

4. Splenium (Posterior)

BLOOD SUPPLY OF INTERNAL CAPSULE


1. ANTERIOR LIMB is supplied by medial striate branches of the
anterior cerebral artery and lateral striate branches of lateral
cerebral artery.

2. GENU by recurrent artery of Huebner or direct branches of the


middle cerebral artery and branches of the anterior choroidal artery.

3. POSTERIOR LIMB by lateral striate branches of the middle


cerebral artery and branches of the anterior choroidal artery.

Page 256 of 284


NAME OF THE BONES MEETING AT PTERION
 The PTERION is the H-shaped formation of sutures on the side of
the calvarium representing the junction of four skull bones:

a. The greater wing of sphenoid bone


b. Squamous portion of the temporal bone
c. Parietal bone
d. Frontal bone

Page 257 of 284


FALX CEREBRI
 The falx cerebri is a large sickle shaped fold of dura mater occupying
the median longitudinal fissure between the two cerebral
hemispheres.

 It has two ends:


a. The anterior end is narrow and is attached to the crista galli.
b. The posterior end is broad and is attached along the median
plane to the upper surface of the tentorium cerebelli.

Page 258 of 284


The falx cerebri has two margins:

1. The upper margin is convex and is attached to the lips of the


sagittal sulcus.
2. The lower margin is concave and free.

 The falx cerebri has right and left surfaces each of which is related to
the medial surface of the corresponding cerebral hemisphere.
 Three important venous sinuses are present in relation to this fold.
 The superior sagittal sinus lies along the upper margin, the inferior
sagittal sinus along the lower margin, and the straight sinus along
the of attachment of the falx to the tentorium cerebelli.

Page 259 of 284


EPICRANIAL APONEUROSIS
 The third layer of scalp is the deep fascia in the form of the
epicranial aponeurosis or gale aponeurotica with the
occipitofrontalis muscle.
 The epicranial aponeurosis is freely movable on the pericranium
along with the overlying and adherent skin and fascia.
 Anteriorly it receives the insertion of the frontalis and posteriorly
it receives the insertion of the occipitalis and is attached to the
external occipital protuberance, and to the highest nuchal lines
in between the occipital bellies.
 On each side, the aponeurosis is attached to the superior
temporal line, but sends down a thin expansion which passes
over the temporal fascia and is attached to the zygomatic arch.

SHOW THE VENOUS SINUSES RELATED TO FALX CEREBRI

Three important venous sinuses are present in relation to this fold. The
superior sagittal sinus lies along the upper margin, the inferior sagittal
sinus along the lower margin, and the straight sinus along the of
attachment of the falx to the tentorium cerebelli.

NAME THE STRUCTURES PASSING THROUGH INTERNAL


ACOUSTIC MEATUS

 Internal acoustic meatus lies in the temporal bone and exists


between the inner ear and posterior cranial fossa.
 It includes the vestibulocochlear nerve (CN VIII), facial nerve (CN
VII), the labyrinthine artery, and the vestibular ganglion.

Page 260 of 284


NAME THE STRUCTURE PASSING THROUGH FORAMEN
SPINOSUM

 The foramen spinosum is located in the posteromedial part of


greater wing of sphenoid bone posterolateral to foramen ovale
which connects the middle cranial fossa with the infratemporal
fossa.
 It transmits the middle meningeal artery, middle meningeal vein,
and the nervus spinosus.

NAME THE STRUCTURES PRESENT IN LATERAL WALL OF


CAVERNOUS SINUS

Structures within the lateral wall from above downwards:

1. OCULOMOTOR NERVE: in the anterior part of the sinus, it


divides into superior and inferior divisions which leave the sinus
by passing through the superior orbital fissure.
2. TROCHLEAR NERVE: in the anterior part of the sinus, it
crosses superficial to the oculomotor nerve, and enters the orbit
through the superior orbital fissure.
3. OPHTHALMIC NERVE: in the anterior part of the sinus, it
divides into the lacrimal, frontal and nasociliary nerves.
4. MAXILLARY NERVE: it leaves the sinus by passing through the
foramen rotundum on its way to the pterygopalatine fossa.
5. TRIGEMINAL GANGLION: the ganglion and its Dural cave
may project into the lateral wall of the sinus.

Page 261 of 284


Page 262 of 284
PTERION
 The pterion is the H-shaped formation of sutures on the side of the
calvarium representing the junction of four skull bones:
a. The greater wing of the sphenoid bone
b. Squamous portion of the temporal bone
c. Frontal bone
d. Parietal bone

 Pterion site in the anterolateral fontanelle is the thin part of skull.


In accidents, the anterior division of middle meningeal artery at
pterion may be ruptured, leading to clot formation between the skull
and dura mater or extradural haemorrhage.
 The clot compresses the motor area of the brain and causes paralysis
in the opposite side of the body.
 The clot must be sucked out at the earliest.

EMISSARY VEIN

1. Emissary veins connect the extra cranial veins with the


intracranial veins.
2. venous sinuses to equalise the pressure.

VEINS
FORAMEN OF VENOUS
NAME OUTSIDE
SKULL SINUS
SKULL

Parietal venous Superior


Parietal foramen Veins of scalp
sinus sagittal sinus

Page 263 of 284


Mastoid venous
Mastoid foramen Veins of scalp Sigmoid sinus
sinus

Internal jugular
Emissary vein Hypoglossal canal Sigmoid sinus
vein

Condylar emissary Posterior condylar Suboccipital


Sigmoid sinus
vein foramen venous plexus

Pharyngeal Cavernous
2- 3 emissary vein Foramen lacerum
venous plexus sinus

Pterygoid venous Cavernous


Emissary vein Foramen ovale
plexus sinus

Veins from the


Superior
Emissary vein Foramen caecum upper part of
sagittal sinus
nose

Page 264 of 284


LIST OUT THE PAIRED DURAL VENOUS
SINUSES
 There are 8 paired Dural venous sinus:
a. Cavernous sinus
b. Superior petrosal sinus
c. Inferior petrosal sinus
d. Transverse sinus
e. Sigmoid sinus
f. Sphenoparietal sinus
g. Petrosquamous sinus
h. Middle meningeal sinus

Page 265 of 284


LIST OUT THE UNPAIRED DURAL VENOUS
SINUSES
 There are 7 unpaired Dural venous sinus:
a. Superior sagittal sinus
b. Inferior sagittal sinus
c. Straight sinus
d. Occipital sinus
e. Anterior intercavernous sinus
f. Posterior intercavernous sinus
g. Basilar plexus of veins

Page 266 of 284


TENTORIUM CEREBELLI
 It is a tent-shaped fold of the dura mater forming the roof of the
posterior cranial fossa.
 It separates the cerebellum from the occipital lobes of the
cerebrum.
 It has two margins and two surfaces.

MARGINS

 The inner free margin is U-shaped and encloses the tentorial


notch for the passage of the midbrain. The anterior ends of the
concave free margin are attached to the anterior clinoid processes.
 The outer attached margin is convex and attached on each side
(from before backward) to the posterior clinoid process, the
posteroinferior angle of the parietal bone, and the lips of
transverse sulci on the occipital bone.
 The free and attached margins cross each other near the apex of
the petrous temporal bone to enclose a triangular area anteriorly
which is pierced by the oculomotor nerve.

SURFACES

 The convex upper surface slopes to either side from the median
plane. In the median plane, it provides attachment to falx cerebri.
 The concave inferior surface provides attachment to falx cerebelli
in its posterior part.

TRIGEMINAL OR MECKEL’S CAVE:

 It is a recess of the dura mater present in relation to the attached


outer margin of the tentorium cerebelli.

Page 267 of 284


 It is formed by the evagination of the meningeal layer of the dura
mater by two roots of the trigeminal nerve below the superior
petrosal sinus.

VENOUS SINUS ENCLOSED IN THE TENTORIUM


CEBRI:

1. TRANSVERSE SINUS within the posterior part of the attached


margin.
2. SUPERIOR PETROSAL SINUS within the anterolateral part of
attached margin.
3. STRAIGHT SINUS within in the line of attachment between the
falx cerebri and tentorium cerbelli.

Page 268 of 284


SUBOCCIPITAL NERVE
 The suboccipital nerve is the dorsal primary ramus of the first
cervical nerve (C1). It exits the spinal cord between the skull and the
first cervical vertebra, the atlas. It lies within the suboccipital
triangle along with the vertebral artery.
 It supplies the four suboccipital muscles and the semispinalis capitis.
 The nerve to the inferior oblique gives off communicating ranch to
the greater occipital nerve capitis.

TRIBUTARIES OF CAVERNOUS SINUS


 The tributaries or incoming channels are as follows:

FROM THE ORBIT:

a. The superior ophthalmic vein


b. A branch of the inferior ophthalmic vein
c. The central vein of retina

FROM THE BRAIN:

a. Superior middle cerebral vein


b. Inferior cerebral vein from the temporal lobe

FROM THE MENINGES:

a. Sphenoparietal sinus
b. The frontal trunk of the middle meningeal vein may drain
directly into the pterygoid plexus through the foramen ovale.

Page 269 of 284


STRUCTURES PASSING THROUGH THE
FORAMEN OVALE
 Foramen ovale transmits the
following structures:
a. The mandibular nerve
b. The accessory meningeal
artery
c. The lesser petrosal nerve
d. An emissary vein
connecting cavernous
sinus with the pterygoid
plexus of veins.
e. Anterior trunk of middle
meningeal artery

LIGAMENTUM DENTICULATUM
 Ligamentum denticulatum are 21 pairs of teeth like projections.
 They fuse laterally with the arachnoid and duramater midway
between the exits of the roots of adjacent spinal nerves.
 The highest process attaches immediately superior to the foramen
magnum.
 The ligamentum denticulate keeps the spinal cord in position.

Page 270 of 284


ARACHNOID GRANULATIONS
 Arachnoid granulations are structures filled with cerebrospinal
fluid (CSF) that extend into the venous sinuses through
openings in the dura mater and allow the drainage of CSF from
subarachnoid space into venous system.
 The lesions are primarily located in the parasagittal region
along the superior sagittal sinus, which is occasionally seen at
the transverse sinus.

CROSS SECTIONS OF SPINAL CORD AT CERVICAL LEVEL

 At the level of cervical vertebrae, the segment of spinal cord is oval


shaped
a. POSTERIOR HORN - slender
b. LATERAL HORN - absent
c. ANTERIOR HORN - narrow in 1-3 segments, broad in C4 -
C8 segments to supply the upper limbs.

Page 271 of 284


CROSS SECTION OF SPINAL CORD AT
THORACIC LEVEL
 At the level of thoracic level, the segment of spinal cord is circular.
 Posterior horn- slender
 Lateral horn- present for thoracolumbar outflow
 Anterior horn- slender in T2- T12 segments, broad in T1 segment.

Page 272 of 284


LUMBAR PUNCTURE
 The lumbar puncture is done to draw CSF from subarachnoid space
for diagnostic and therapeutic purposes.
 The needle is inserted between L3 and L4 or L4 and L5 vertebrae
with patient’s back flexed either in sitting position or lying on a bed
in left lateral position, usually the later when patient is curled up
lying on left side.
 The needle passes through the supraspinous and interspinous
ligaments and between ligamenta flava before piercing the dura
mater. As the dura mater is pierced, there is a distinct feel of give
way.
 Since the spinal cord ends at the lower border of L1 vertebra, there is
no danger of damage to the spinal cord

NERVES RISING FROM MEDULLA OBLONGATA


 Rootlets of the hypoglossal nerve: These emerge from the
anterolateral sulcus between the pyramid and the olive.
 Rootlets of the 9th, 10th, and 11th (cranial part) cranial nerves:
These emerge through the posterolateral sulcus separating the olive
from the inferior cerebellar peduncle.

Page 273 of 284


LIST SOMATIC AFFERENT NUCLEI
 General somatic afferent (GSA) fibres.
 They carry general sensations of pain, touch, and temperature
from skin and proprioceptive sensations of vibration and muscle
and joint sense.

MODIFICATIONS OF CRANIAL PIA MATTER


 The pia mater provides sheath for the cranial nerves merging with
the epineurium around them.
 It also provides perivascular sheath for the minute vessels
entering around them.
 Folds of pia mater enclosing the tufts of capillaries form the
telanchoroidea. Such pia mater lined by secretary ependyma from
the choroidal plexus.

TERMINAL BRANCHES OF FACIAL NERVE


 Five terminal branches
1. Temporal
2. Zygomatic
3. Buccal – upper
buccal and lower
buccal nerves
4. Marginal mandibular
nerve
5. Cervical nerve

Page 274 of 284


INDICATE THE PALEO CEREBELLAR DEEP
NUCLEI
 Globose nucleus and Emboliform nucleus are the paleo cerebellar
nuclei
 They are collectively called as Nucleus interpositus

DEEP NUCLEI OF CEREBELLUM (ALSO CALLED


AS INTRACEBELLAR NUCLEI OR CENTRAL
NUCLEI)
 From lateral to medial
a. DENTATE NUCLEUS - Nucleus of neocerebellum; largest
nucleus
b. EMBOLIFORM NUCLEUS - Nucleus of paleocerebellum
c. GLOBOSE NUCLEUS - Nucleus of paleocerebellum
d. FASTIGIAL NUCLEUS - Nucleus of archicerebellum

LIST SPECIAL SOMATIC AFFERENT NUCLEI


1. COCHLEAR NUCLEI - Dorsal, ventral → Receives fibres from
spiral ganglion
2. VESTIBULAR NUCLEI - Medial, lateral, inferior, superior
→Receives fibres from cerebellum

Page 275 of 284


MIDDLE MENINGEAL ARTERY
 Branch of first part of Maxillary artery.
 Passes through foramen spinosum, enters cranial cavity, supply
meninges.
 Divides into anterior and posterior branches in cranial cavity.
 Anterior branch crosses pterion and posterior branch ascends
backwards towards lambda.
 In skull bone fracture or head injury this artery may get ruptured
deep to pterion leading to EXTRADURAL HEMATOMA.

PINEAL GLAND
 Neuroendocrine gland secreting melatonin.
 Situated in a vertical groove between 2 superior colliculi below
splenium of corpus callosum.
 The stalk has two laminae-
a. SUPERIOR -continues with habenular commissure.
b. INFERIOR-continues with posterior commissure.
 Calcium phosphates and carbonates gets deposited in pineal gland
with age from of multilaminar corpuscles -called BRAIN SAND OR
CORPORA ARENACEA.
 Pineal gland is supplied by nervus conarii.
 It acts as a biological clock.
 Pineal gland is the only part of brain, which has no nerve cells in it.

Page 276 of 284


ORIGIN AND BRANCHES OF MIDDLE
MENINGEAL ARTERY
ORIGIN:

 Largest meningeal branch -first part of Maxillary artery.


 Ascends up into cranial cavity via foramen spinosum.

BRANCHES: Anterior frontal branch-

 Courses up towards pterion, then curves downward ascend towards


cortex.
 In pterion region, middle meningeal artery frequently lies on bony
tunnel in parietal bone.

POSTERIOR PARIETAL BRANCH-

 Arch backwards on squamous part of temporal bone.


 Crosses lower border of parietal bone in front of mastoid angle.
 Gives branches that supplies posterior part of dura mater and
cranium.

Page 277 of 284


FORMATION OF BASAL VEIN
 Formed at base of brain in the region of anterior perforated
substance by union of 3 veins
a. ANTERIOR CEREBRAL VEIN
b. DEEP MIDDLE CEREBRAL VEIN
c. STRIATE VEINS-from anterior perforated substance
 Runs posteriorly around midbrain, medial to uncus,
parahippocampus.
 Terminates in Greater cerebral vein (vein of Galen) below
splenium of corpus callosum.
 Basal vein is also called “VEIN OF ROSENTHAL”.

BASILAR ARTERY
 Formed by union of 2 vertebral arteries at the lower border of pons.
 Basilar artery ascends in midline on ventral surface of pons and at
its upper border terminates into right and left posterior cerebral
arteries

BRANCHES:

a. Pontine branches
b. Anterior inferior Cerebellar arteries
c. Labyrinthine artery
d. Superior Cerebellar artery
e. Posterior cerebral arteries

Page 278 of 284


CORPUS STRIATUM
 Located lateral to thalamus.
 Part of basal ganglia.
 Includes caudate nucleus and lentiform nucleus.

STRUCTURES FORMING LIMBIC SYSTEM

CORTICAL STRUCTURES-

a. LIMBIC LOBE- Cingulate gyrus, isthmus,


Parahippocampal gyrus, uncus
b. HIPPOCAMPAL FORMATION- hippocampus, dentate
gyrus, gyrus fasciolaris, indusium griseum.

Page 279 of 284


SUBCORTICAL NUCLEI

1. Amygdaloid body
2. Septal region and nuclei
3. Olfactory areas
4. Mammillary bodies
5. Anterior nucleus of thalamus

FIBRE BUNDLES

1. Fornix
2. Mammillothalamic trac
3. Stria terminalis
4. Anterior commissure
5. Stria medullaris thalami
6. Median forebrain bundle
7. Cingulum
8. Diagonal band of Broca

COMPONENTS OF BASAL GANGLIA


1. Corpus striatum (caudate nucleus and lentiform nucleus)
2. Claustrum
3. Amygdaloid body
4. Substantia nigra, red nucleus
5. Subthalamic nucleus

Page 280 of 284


PARKINSONISM
 Also called Parkinson’s disease or paralysis agitans
 Due to deficiency of dopamine in corpus striatum
 Due to lesion in Substantia nigra or nigrostriate fibres

FEATURES:

a. Resting tremors
b. Lead pipe muscular rigidity
c. Mask-like face (no facial expression)
d. Shuffling gait
e. Pill rolling movements of hand
f. Absence of associated movements

FUNCTIONAL AREAS OF SUPERIOR


TEMPORAL GYRUS
 PRIMARY AUDITORY AREA (Brodmann’s area 41 and 42) located
in superior surface of superior temporal gyrus occupying anterior
transverse temporal gyrus (Heschl's gyrus)
 SECONDARY AUDITORY AREA (Brodmann’s area 22) lateral
surface of superior temporal gyrus posterior to primary auditory
area.
 Receives fibres from medial geniculate body via auditory radiation.
 Cochlea is bilaterally represented.
 Lesion in one cortex does not cause unilateral deafness.

Page 281 of 284


SENSORY THALAMIC NUCLEI
 Medial geniculate body -auditory
 Lateral geniculate body-visual
 Ventral posterior nucleus -Somatosensory, vestibular

INSULA
 Insula is the submerged /hidden portion of cerebral cortex in floor of
lateral sulcus
 Triangle in shape surrounded by sulcus, the circular sulcus except at
the apex.
 It can be seen only when lips of lateral sulcus are widely pulled apart
 During brain development, due to overgrowth of surrounding cortical
areas it is submerged
 Also called ISLAND OF REAL or CENTRAL LOBE or FIFTH
LOBE
 It’s is divided into anterior and posterior region by Central sulcus.
 Anterior region presents 3 or 4 short gyri called gyri brevia and
posterior region presents long gyri called gyri longa.

VISUAL STRIA
 It is also called Line of Gennari, it refers to the primary visual
cortex, Broadman's area No-17, located in the occipital Lobe.

Page 282 of 284


STRUCTURES LODGED IN LATERAL SULCUS
OF CEREBRUM
a. Superficial Middle cerebral vein

b. Left lateral cerebral artery with its superficial and inferior


terminal branches.

PARTS OF CAUDATE NUCLEUS


1. Head-Rounded anterior part in front of interventricular foramen

2. Body

3. Tail-merges at its anterior extremity with an almond shaped mass


of grey matter-amygdaloid body

CORPUS CALLOSUM
 It is the largest of the commissural fibres (interhemispheric
fibres).

 It consists of about 100 million fibres.

 It is largely responsible for interhemispheric transfer of


information which is essential for bilateral responses and learning
processes.

Page 283 of 284


PARTS OF CORPUS CALLOSUM
1. Rostrum (Anterior)

2. Genu

3. Body/Trunk (Middle part)

4. Splenium (Posterior)

BLOOD SUPPLY OF INTERNAL CAPSULE


1. ANTERIOR LIMB is supplied by medial striate branches of the
anterior cerebral artery and lateral striate branches of lateral
cerebral artery.

2. GENU by recurrent artery of Huebner or direct branches of the


middle cerebral artery and branches of the anterior choroidal
artery.

3. POSTERIOR LIMB by lateral striate branches of the middle


cerebral artery and branches of the anterior choroidal artery.

Page 284 of 284

You might also like